You are on page 1of 64

Physical Sciences Solutions for AAMC 3R

1. Reaction 4 is shown in the following equation, which is answer choice A.

PbCO3(s) + 2 HCl(aq) PbCl2(aq) + CO2(g) + H2O(l)

Answers B and D do not show a reaction involving PbCO3(s), as required by Reaction 4. Answer C shows
an implausible and unbalanced equation. Thus, answer choice A is the best answer.
2. Reaction 1 is shown in the following equation.

Pb(NO3)2(aq) + Na2SO4(aq) PbSO4(s) + 2 NaNO3(aq)

Compound A, the white solid, is PbSO4(s). Neither the reactant Pb(NO3)2 nor the product NaNO3 can
precipitate because all nitrates and sodium salts are water soluble. PbI2 cannot precipitate because iodide
is not present. Thus, answer choice D is the best answer.
3.

The dissolution of Pb(OH)2(s) is represented by the following equation.

Pb(OH)2(s)

Pb2(aq) + 2 OH-(aq)

At pH 9, the concentration of OH-(aq) is greater than the concentration of OH-(aq) at pH 7. According to


Le Chtelier's principle, the additional common ion, OH-(aq), will shift the position of equilibrium to the
left, and less Pb(OH)2 will dissolve. Thus, answer choice A is the best answer.
4. The reactions described in the passage show that lead(II) is successively precipitated as PbSO4,
PbI2, and PbCO3. This sequence shows (assuming equal anion concentrations, as must be done
here) that PbCO3 is less soluble than PbI2, and PbI2 is less soluble than PbSO4. The order in which
the anions precipitate Pb2+ is: CO32- then I- then SO42-. When this sequence is applied to the
question, answer choice B is in the correct order, and answers A, C, and D are all in the opposite
order. Thus, answer choice B is the best answer.

Page 1 of 64

5. The initial Na2SO4(aq) solution in Reaction 1 is 15 mL of 0.300 MNa2SO4(aq).

(15.0 mL) ( 1 L ) (0.300 mol Na2SO4) (2 mol Na+)


(1000 mL) (1 L Na2SO4(aq) ) (1 mol Na2SO4)

= 0.00900 mol = Answer B


6. The liquid and vapor phases coalesce at point D of Figure 2, where the densities of liquid and
gaseous CO2 are equal. Thus, answer choice D is the best answer.

7. The question does not compare CO2 to a specific solvent, so we are looking for an inherent
property of CO2 that makes it a good solvent for an organic oil. Supercritical CO2 is similar to a
liquid and can be easily removed by evaporation because it changes into a gas when the pressure
is lowered. Answers A, B, and C are not true of CO2, and answers A and C are not desirable
properties of an extraction solvent. Thus, answer choice D is the best answer.

8.
Polar water molecules are held together by relatively strong hydrogen bonds; whereas, the linear,
nonpolar molecules of CO2 are held together at room temperature by weak London dispersion
forces. Thus, answer choice D is the best answer.

9. The critical point, shown as a dot () in Figure 1, is near 30oC and 80 atm. Answer choice C is the
best answer.

10. According to the principle of like dissolves like, the covalent compound CO2 is a better solvent
for a covalent compound than it is for an ionic compound. Diethyl ether, C2H5OC2H5, is a
covalent compound and NaCl, NH4NO3, and KOH are ionic compounds. Thus, answer choice B
is the best answer.

11. The emission peaks P1 and P2 are described in the passage as due to an electron from an outer
energy level filling a vacant inner energy level, resulting in emission of an X-ray photon. These
photons have discrete energies, and therefore discrete wavelengths, so they appear in the
spectrum as peaks. Thus, answer choice C is the best answer.

Page 2 of 64

12. The power P, supplied by the battery to accelerate the electron beam is given by the formula P =
IV, where I is the beam current and V is the potential difference between the cathode and anode.
Therefore P = (5 x 10-3 A) x (105 V) = 5 x 102 W. Therefore, answer choice A is the best answer.

13. The emitted X-ray has the positive energy difference between the atomic energy levels as
measured by their ionization potentials. For Pb n = 2 to 1, that is (1,400 x 10-17 J) - (240 x 10-17 J)
= 1160 x 10-17 J = 1.16 x 10-14 J. Thus, answer choice C is the best answer.

14. According to the passage, bremsstrahlung is produced when electrons are accelerated during
collisions with ions. All the choices of answers are ions except He, a neutral atom. Therefore,
answer choice A is the correct answer.

15. To increase the kinetic energy of the electrons, they must be accelerated by a higher voltage
between the cathode and anode, thus the voltage of HV was increased. Thus, answer choice A is
the best answer.

16. The probability of an X-ray emission event at a given wavelength is measured by its intensity in
the spectrum. In Figure 2, P2 has a higher intensity than does P1. Thus, answer choice D is the
best answer.

17. The dipole moment of a molecule is the vector sum of all of the bond moments. According to the
data in Table 1, the dipole moment of SnBr4 is zero; therefore, its bond moments add to zero or
cancel. Thus, answer choice D is the best answer.

18.
The 2s electron cloud in NO is in a bonding molecular orbital (MO) that forms by the overlap of
the 2s orbital of an oxygen atom with the 2s orbital of a nitrogen atom. Because O is more
electronegative than N, the electron cloud in the resulting 2s MO is larger around the oxygen
atom than it is around the nitrogen atom. Thus, answer choice D is the best answer.

19. Table 1 gives the dipole moment of HF as 1.82 D. Chlorine is just below fluorine in the periodic
table; therefore, the electronegativity of chlorine, though significant, is less than that of fluorine.
Chlorine is less effective than fluorine in creating a separation of charge when bonded to
hydrogen, and the dipole moment of HCl is slightly less than that of HF. Thus, answer choice B is
the best answer.

Page 3 of 64

20. HCl is polar covalent because H and Cl share a pair of bonded electrons that are more strongly
attracted to the chlorine atom. The higher effective nuclear charge (i.e., the charge of the nucleus
minus the shielding caused by extranuclear electrons) of chlorine accounts for its greater
electronegativity. Thus, answer choice D is the best answer.

21. Carbon dioxide,O=C=O, is linear. Therefore, the two CO dipoles cancel because they are in
opposite directions. If one of the oxygen atoms is removed, the resulting CO will have a dipole
because the species is linear and comprised of two different atoms. Thus, the dipole moment will
change from zero in CO2 to a positive value in CO. Thus, answer choice D is the best answer.

22. An analysis of the two structures shows that the bond moments in PCl5 add to zero; whereas,
those in PCl3 do not. As shown in the figure, PCl3 is pyramidal not planar. Thus, answer choice B
is the best answer.

23. Work is the product of the force on an object and the distance the object moves in the direction of
the applied force. In this case, work = 20 N x 10 m = 200 J. Thus, answer choice C is the best
answer.

24. Evaporation occurs when a molecule attains sufficient speed or kinetic energy to overcome the
attractive forces of a liquid. Resonance, surface tension and potential energy all relate to
molecules that are not in motion. Thus, answer choice B is the best answer.

25. The relation between distance, acceleration, and time is: d = (1/2)a t2. To solve for the time it
takes the runner to use t = (2d/a)1/2 = (23/1.5)1/2 = 2 s. Therefore, answer choice C is the best
answer

26. A body is in transitional equilibrium when the components of all external forces cancel. For the
sheet: F cos = 4 N, F sin = 3 N. The magnitude of F is found by adding the squares of the
components: F2cos2 + F2 sin2 = F2 = 42 + 32 = 25 N2. Therefore F = 5 N. The F vector points
in the proper direction since tan = 0.75 = 3/4. Thus, answer choice C is the best answer.

Page 4 of 64

27. The only experimental difference in Trial 1 vs. Trial 2 is that, in Trial 2, the test tube is placed in
water (20oC) to cool rather than in air (also 20oC). In other words, only the surroundings were
different. Thus, answer choice B is the best answer.

28. The melting point of acetamide is 80oC; therefore, acetamide will melt when it is in a test tube
that is placed in a water bath at 90oC. The temperature of the water in the bath, not the amount of
water in the bath, determines whether or not the acetamide will melt. The period of time for
acetamide to melt, starting at 90oC, is more than the corresponding period, starting at 100oC (i.e.,
the temperature of boiling water). Thus, answer choice D is the best answer.

29. Without controlling the temperature (i.e., raising the temperature of the water bath above 80oC),
the experimenter could not have observed melting or freezing. Without monitoring the time, the
experimenter could not have determined the period of time for the samples to melt or freeze. The
temperature of melting (freezing) of a pure substance such as acetamide is independent of the
amount melted, and Experiment 2 shows that the surroundings control the period of time for
freezing to occur. Thus, answer choice A is the best answer.

30.
The time period of melting is independent of the time intervals used by the experimenter to record
temperatures. The sample would freeze completely after 23 min regardless of the time interval
used by the experimenter to record temperatures. Thus, answer choice B is the best answer.

31. After Experiment 1, the sample was removed from a hot water bath as a liquid. Subsequently, the
sample froze during Trial 1. Therefore, the sample had to be reheated in a water bath above its
melting point to start Trial 2 as a liquid. Thus, answer D is the best answer.

32. If the data for Trial 1 were plotted, the temperature would drop to 80oC and remain at this melting
temperature for 23 min (or 23 min x 60 sec/min = 1380 sec). The line at 80oC would not slope
downward at all in the figure, and it would extend well past 270 sec, the maximum time shown in
the figure. Thus, answer choice A is the best answer.

33.
The tone with the shortest period has the shortest wavelength. In Figure 1a, the period of the third
harmonic (the curve with the smaller dashes) is seen to be shorter than the other two harmonics.
Thus, answer choice C is the best answer.

Page 5 of 64

34. The three curves in Figure 1a intersect at three points in time. The second intersection occurs in
the middle of the time axis. At that point all three curves have zero displacement. Therefore,
answer choice C is the best answer.

35. The period T and frequency f of a tone are related by T = 1/f. If the first harmonic has a frequency
of 100 Hz, then the second harmonic has a frequency of 200 Hz. The period corresponding to 200
Hz is 1/200 s-1 = 0.005 s. Thus, answer choice A is the best answer.

36. The amplitudes of the three harmonics can be compared in Figure 1a. The first harmonic is seen
to be largest while the other two have equal amplitudes. Answer choice A best represents these
observations.

37. A fourth harmonic would have a shorter period than the other three. Since T = 1/f, the fourth
harmonic would have a higher frequency than the third harmonic. Therefore, answer choice D is
the best answer.

38. The waveform in Figure 1c begins to repeat at the zero displacement point near the end of the
time axis. This is the same time period as the first harmonic as seen in Figure 1a. Thus, answer
choice A is the best answer.

39. The relation of wavelength, frequency, and wave velocity is f = v. For light, v = 3 x 108 m/s.
The wavelength for Material B at an efficiency of 0.42 is read from Table 1 as = 1.06 x 10-6 m.
The frequency of this light is 3 x 108 m/s divided by 1.06 x 10-6 m, giving 2.8 x 1014 Hz. Thus,
answer choice C is the best answer.

40. The passage states "a coating that maximized the absorption of light." Therefore, the coating also
maximizes the conversion efficiency. Thus answer choice C is the best answer.

41. Identical voltage sources connected in parallel produce the same output voltage as a single
source. (Whereas if they were connected in series, the source voltages would be added.)
Therefore, answer choice D is the best answer.

Page 6 of 64

42. The equation K = hf given in the passage implies that the photon energy must be greater than
the work function of the material in order to liberate an electron, i.e. for K to be positive. Thus,
answer choice A is the best choice.

43. Applying a coating that makes independent of the wavelength means is the same for all .
Thus, is constant when plotted versus , a horizontal line. Therefore, answer choice C is the best
answer.

44. According to the data in Table 1, both structure and molecular weight (i.e., molar mass) affect the
melting point of a compound. In order to assess the effect of molecular weight or mass alone, any
other effects such as obvious structural differences must be minimized. This is best done by
comparing two compounds that are structurally similar. Because the structures of propionic acid
and butyric acid (Answer C) differ by only a CH2 group, they best show that melting point
increases with molar mass. All of the other answer choices compare two compounds that differ
significantly in structure. Therefore, the melting points of these compounds include both molar
mass and structural effects. Thus, answer choice C is the best answer.

45. Figure 1 shows the pH of the solution to be about 3 before any NaOH(aq) is added.

pH = -log[H3O+]
3 = -log[H3O+]
+
-3
[H3O ] = 10 M = 0.001 M = Answer A
46. The freezing point depression of an aqueous solution is a colligative property (i.e., it depends on
the number of solute particles in a given volume of water.) Given two solutions, the one with the
greater number of solute particles per liter of solution freezes at the lower temperature. Answer C
is the only answer that relates a larger number of solute particles directly to a lower freezing
point. Oxalic acid is diprotic and ionizes in accord with the pKa values in Table 1 to a greater
extent than does crotonic acid. Subsequently, oxalic acid requires more NaOH than does crotonic
acid to reach a pH of 4.7, and oxalic acid produces a larger number of particles in solution. Thus,
answer choice C is the best answer.

47. In a titration of RCOOH, the concentrations of RCOOH and RCOO- are equal at the midpoint of the titration. This is often called the half-equivalence point. From the expression for the
equilibrium constant of a weak acid HA, when [HA] = [A-], then [H3O+] = Ka and pH = pKa.
Table 1 shows the pKa value for a monoprotic acid to be 4.694.88. Answer choice A (4.8) lies in
this range, the other choices do not. Alternatively, Figure 1 shows the pH at the half equivalence
point of a weak acid to be about 4.8. Thus, answer choice A is the best answer.

Page 7 of 64

48. The first sentence of the passage states that the unknown was a liquid at room temperature
(20oC). Table 1 shows that the melting point of crotonic acid is 71.6oC, which means it is a solid
at room temperature (i.e., it melts 51.6oC above room temperature). Thus, answer choice C is the
best answer.

49. In general, catalysts lower the activation energy of the slowest step in a reaction. Thus, they
increase the rate of the reaction without increasing the number of collisions, the kinetic energy of
the reactants, or the Keq of a reversible reaction. Thus, answer choice C is the best answer.

50. Conservation of linear momentum requires: mradonvradon= mheliumvhelium with helium identified as the
alpha particle. The nuclear masses can be approximated by their mass numbers (222 and 4). Thus,
the recoil speed of the radon is (4/222)1.5 x 107 m/s = 2.7 x 105 m/s. Therefore, answer choice B
is the best answer.

51. The overall order of a reaction is the sum of the exponents of the concentrations in the rate law.
The exponent of [NO2] is 1 and that of [F2] is 1, and their sum is 1 + 1 = 2. Thus, the overall order
is two or second order. Thus, answer choice C is the best answer.

52. The problem gives Hfo for HCl as -92.5 kJ/mol. This means that the formation of one mole of
HCl from its elements liberates 92.5 kJ of heat, as shown in the following equation.

1/2 H2(g) + 1/2 Cl2(g) HCl Hfo = -92.5 kJ

Therefore, the formation of two moles of HCl liberates twice this amount or 185.0 kJ.

H2(g) + Cl2(g) 2 HCl H = -185.0 kJ

The question asks for the enthalpy change (H) for the reverse reaction. When the reaction is
reversed, the sign of H is changed from to +. Thus, the reverse reaction requires +185.0 kJ =
Answer D.

Page 8 of 64

53. The ratio of object to image distance equals the ratio of object to image height. The ratio of image
to object height is found by rearranging the ratios to give 4f /(4/3)f = 1/3. The image is
demagnified by a factor of 3. Thus, answer choice A is the best answer.
54. In the artery, the product of blood speed and artery cross-sectional area is everywhere constant
because the volume flow rates are equal. At the point with half the normal area, the speed must
double so that the same volume of blood passes through the constriction as does through the
normal part of the artery. Thus, answer choice C is the best answer.

55. A stronger B field increases the magnetic force, Fm = q v B. The electric force must also
increase to achieve equilibrium. This implies a larger electric field in the artery and a larger
voltage across the artery. Thus, answer choice C is the best answer.

56. Volume flow rate is the product of blood speed and artery cross-sectional area: (0.20
m/s)(/4)(1.0 x 10-2 m)2 = 5 x 10-6 m3/s. Therefore, answer choice B is the best answer.

57. Since volume flow rate is proportional to blood speed, it doubles when v doubles. Therefore,
answer choice B is the best answer.

58. A magnetic force acts on a moving charge in a direction that is perpendicular to both the velocity
of the charge and the direction of the magnetic field. This is a basic law of the interaction of
electric currents and magnetic fields. Thus, answer choice D is the best answer.

59. The passage states that sulfuric acid reacts with Cu(s) to produce Cu+ and SO2. Thus, sulfuric acid
is converted into sulfur dioxide, or H2SO4 SO2. The oxidation number of sulfur in H2SO4 can
be found by assigning oxidation numbers of +1 for hydrogen and -2 for oxygen. For the formula
H2SO4 to be neutral, the sum of the oxidation numbers must be zero. If x is the oxidation number
of sulfur in H2SO4, then: 2(1) + 4(- 2) + x = 0, and x = +6. Likewise, for SO2: 2(- 2) + x = 0, and x
= +4. The change in oxidation number is from +6 to +4, which is answer choice B.

60. The boiling point of HNO3 is given in the question as 86oC. Because HNO3 must boil out of the
flask and be trapped in the tube, the temperature of the flask must be above the boiling point of
HNO3 (i.e., < 86oC) and the temperature of the tube must be less than the boiling point of HNO3
(i.e, < 86oC). Answer B meets these criteria; the other answers do not. Thus, answer choice B is
the best answer.

Page 9 of 64

61. The combustion of elemental sulfur involves a reaction between oxygen (O2) and sulfur (S).
[Note: Though sulfur exists as S8 molecules, its reactions are normally written in terms of its
empirical formula S.] Only Answer D shows such a reaction. Thus, answer choice D is the best
answer.

62. The reaction involves the formation of gaseous SO3 from gaseous O2 and gaseous SO2.

O2(g) + 2 SO2(g) 2 SO3(g)

According to Le Chtelier's principle, any action that causes the reaction to shift toward the right
will cause O2 and SO2 to react and increase the yield of SO3. Of the four possible actions (answers
A-D), the removal of SO3 as it forms will shift the reaction toward the right and is the most likely
action to increase the yield of SO3. Thus, answer C is the best answer.
63. The passage states that concentrated H2SO4 is 98% H2SO4 and 2% water by mass and that the
density of concentrated H2SO4 is 1.84 g/mL.

64. The problem hypothesizes the liberation of hydrogen in accord with the unbalanced equation
below.

Fe(s) + H2SO4 H2(g)

In the conversion, hydrogen goes from an oxidation state of +1 in sulfuric acid to 0 (zero) in H2.
Thus, the hydrogen in H2SO4 is reduced (i.e., it undergoes an algebraic decrease in oxidation
state). Fe(s) is the reducing agent (i.e., it causes the reduction). Thus, answer choice A is the best
answer.
65. The first ionization of sulfuric acid, H2SO4, is normally 100% in water. However, under
conditions of low water content, all of the H2SO4 cannot ionize. Qualitatively, the only source of
SO42- is HSO4-. The Ka for the second ionization step of a parent acid is a few orders of magnitude
smaller than that of the first step; therefore, SO42- must be the least abundant species, because it is
only formed in the second ionization step. Thus, answer choice A is the best answer.
Quantitatively, the mass relationship in 100 g of 98% H2SO4 is 98 g H2SO4 and 2 g H2O. The
molar mass of H2SO4 is 98 g/mol and of water is 18 g/mol. Thus, in 100 g of 98% H2SO4 there is
one mole of H2SO4 and 2/18 or 1/9 mole of H2O. In excess water, H2SO4 would ionize
completely. However, in this case (i.e., very low water content), only about 1/9 of a mole can

Page 10 of 64

react stoichiometrically with water to form H3O+ and HSO4-. Of this 1/9 mol, only a small
fraction of the HSO4- further ionizes to H3O+ and SO42-, because HSO42- is a weak acid. Therefore,
SO42- is the chemical with the lowest concentration. Thus, answer choice A is the best answer.
66. The reflected radar signal has a frequency shift due to the Doppler effect. Mercury must have
been moving toward the Earth for the shift to be to a higher frequency. Thus, answer choice A is
the best answer.

67. An equatorial bulge of the Sun would be caused by some process that is not the same in all
directions. Of the four answers, only the Sun's rotation about an axis fulfills this condition. The
bulge is related to the balance between the centripetal force and the gravitational force. Therefore,
answer choice A is the best answer.

68. The perihelion will have moved through an angle = t after a time t has elapsed. The
conversion of angles is given by the following. An arcsecond is 1/60 of an arcminute. An
arcminute is 1/60 of a degree. The time for 500 arcseconds per century to accumulate to = 360
degrees is 360/(500/(60 x 60)) = 360 x 60 x (60/500) centuries. Thus, answer choice C is the
correct answer.

69. The orbit of Mercury around the Sun is, by Kepler's and Newton's laws, an ellipse with the Sun at
one fixed focus. Therefore, answer choice C is the best answer.

70. The time for the radar signal to travel from the Earth to Venus and return is the total distance
divided by the velocity of light: 2 x 5 x 1010 m / 3 x 108 m/s = 3.3 x 102s 300 s. Thus, answer
choice B is the correct answer.

71. The ellipse precesses around the Sun at one focus. Any fixed point on the ellipse is a constant
distance from the focus. Therefore it traces out a circle around the focus. Answer choice A is the
best answer.

72. As written, the equation shows the reduction of Zn2+ to Zn. The negative reduction potential for
Zn2+ means that Zn has a positive oxidation potential (i.e., Zn is easily oxidized.) Zinc will
displace hydrogen, which has a zero standard reduction or oxidation potential (i.e., hydrogen is
the standard against which other substances are measured.). As given in the problem, ionic zinc is
Zn2+. Therefore, zinc metal liberates hydrogen gas and produces ZnCl2(aq). Answer choice B is
the best answer.

Page 11 of 64

73. Power is defined as the rate of doing work. For the automobile, the power output is the amount of
work done (overcoming friction) divided by the length of time in which the work was done.
Therefore, answer choice D is the best answer.

74. The bonds labeled C and D in the figure are of equal length but shorter than bond B. This is
because two resonance structures can be drawn: in one resonance structure, bond C is a double
bond, and in the second resonance structure, bond D is double bond. Double bonds (bond order =
2) are shorter than single bonds (bond order = 1), so the bond order for bonds C and D is about
1.5 and for bond B about 1. Bond B is longer than bond A because of the small atomic radius of
hydrogen compared to nitrogen. Thus, answer choice B is the best answer.
Alternatively, the figure shows that C = D, so answer choices C and D can be eliminated. Also, A
is clearly shorter than B; therefore B is the longest. Thus, answer choice B is the best answer.
75. Conservation of energy requires that the 15.0 eV photon energy first provides the ionization
energy to unbind the electron, and then allows any excess energy to become the electron's kinetic
energy. The kinetic energy is this case is 15.0 eV-13.6 eV = 1.4 eV. Thus the correct answer is A.

76. In radioactive decay, the sum of the mass numbers A and atomic numbers Z, before and after
decay, must balance. The numbers for beryllium undergoing positron decay are: mass (7 = 7 + 0)
and atomic (4 = 3 + 1). The resulting nucleus is 73Li. Thus, answer choice B is the best answer.

77. Work is the product of force and distance. The easiest way to calculate the work in this pulley
problem is to multiply the net force on the weight mg by the distance it is raised: 4 kg x 10 m/s2 x
5 m = 200 J. Therefore, answer choice D is the best answer.

Page 12 of 64

Verbal Reasoning Solutions for AAMC 3R

78. According to one authority on the drug problem, "tobacco shortens one's life, cocaine
debases it; nicotine alters one's habits, cocaine alters one's soul." This authority would
probably:
A) support the legalization of cocaine.
The argument presented for the legalization of cocaine requires the assumption that it is
not so dangerous as is commonly believed. Someone who believes that it debases life
and alters one's soul is unlikely to accept this argument.
B) approve the abolition of both cigarette and cocaine sales in the U.S.
The authority contrasts the effects of tobacco and cocaine in order to emphasize the
greater devastation caused by cocaine. The faults of tobacco (shortening life and altering
habits) are equally true of many behaviors e.g., overeating, being sedentary that are
universally regarded as permissible personal choices. The authority is therefore unlikely
to favor the abolition of cigarettes.
C) not consider either cocaine or tobacco particularly dangerous.
Although one might argue that the debasing of ones life and altering of ones soul make
cocaine dangerous in a vague, spiritual sense of the word, the shortening of ones life
signifies that tobacco is physically dangerous.
D) consider both cocaine and tobacco to be harmful but cocaine more so.
The information that introduces the question does not imply agreement between the
authority quoted and the passage author, so the correct response must be found in the
quotation. Although the observations about tobacco constitute a warning it shortens
lives and alters habits they are factual. In contrast, the expressions used about cocaine
are moral judgments it debases lives and alters souls indicating a stronger
condemnation.

79. The author of the passage would probably support most strongly a federal law that:
A) requires mandatory drug-treatment programs for convicted drug users.
If the authors recommendations were adopted, there would be no convictions for
Solution
drug use and nobody may be compelled to undergo treatment for engaging in a
legal activity. Furthermore, the passage stresses the budgetary advantages of drug
legalization, so the author is unlikely to advocate government spending for a
program requiring the services of physicians and other professionals, which would
Guess
undoubtedly be quite costly, even if the treatment were voluntary.

Page 13 of 64

B) grants tax-exempt status to income earned in the drug trade.


Besides the fact that the author advocates the decriminalization of drug use and not
the active promotion of it, a tax advantage for drug-related profits would eliminate
the anticipated new tax revenue from legal drug production and sales . . . of at
least ten billion dollars a year, one of the authors arguments for drug
legalization.
C) dispenses, free of charge, federally certified sterile needles to addicts.
The free distribution of needles would accord with legalization scenarios in at
least three ways. (1) It would serve the goal of enhancing the quality of urban
life by reducing the public-health risk from unsterile needles. (2) It would
facilitate the investigation of any remaining drug-related crime by providing a
means of identifying and monitoring addicts. (3) It would provide data to test one
premise of drug legalization (which would be a prerequisite for a needledistribution program) that repealing many of the drug laws would not lead . . .
to a dramatic rise in drug abuse.
D) assigns to the military the task of intercepting drugs from foreign nations.
If drugs were legal, as the author recommends, their importation would
presumably no longer be subject to extraordinary restrictions.

80. Which of the following claims is/are explicitly presented in the passage to justify the
supposition that public treasuries would enjoy a net benefit as a result of drug
legalization?
Solution

Guess

I.
II.
III.

Income earned from the drug trade would be subject to taxation.


Fewer law-enforcement personnel would be retained.
Fewer public funds would be spent on drug enforcement.

A) I only
Option I is correct, but so is option III.
B) I and II only
Option II is neither stated nor implied. Drug legalization would presumably allow
the law-enforcement personnel assigned to drug-enforcement activities to be
reassigned.

Page 14 of 64

C) I and III only


Between reduced government expenditures on enforcing drug laws [option III]
and new tax revenue from legal drug production and sales [option I], public
treasuries would enjoy a net benefit.
D) I, II, and III
See rationale B.

81. Which of the following findings best supports the author's belief that drug
legalization would not result in a dramatic increase in drug abuse?
A) Most Americans are currently hostile to the idea of drug legalization.
Solution
General hostility toward drug legalization indicates nothing about the probable
effect of legalization on the prevalence of drug abuse.
Guess

B) Most Americans are unlikely to engage in an obviously dangerous activity.


Since drug taking involves well known risks (overdosing, psychosis, disease from
contaminated needles, loss of employment, disintegration of personal
relationships, etc.), evidence that Americans tend to avoid risk-taking activities
supports the argument for legalization.
C) Most Americans do not take the legal status of a substance into account when
deciding whether or not to ingest it.
In the argument that the legal status of cocaine or heroin is irrelevant because few
Americans would choose to inject it into their veins, the implied deterrent is the
riskiness of such behavior. This argument does not pertain to either other currently
illegal drugs or other means of ingestion. The use of such substances may be
restricted by their price, for example, rather than by their legal status. And since
legalization could be expected to make all street drugs more affordable, it might
well promote drug abuse among current nonusers for whom expense is a major
consideration.
D) The consumption of alcohol rose after the repeal of prohibition.
Evidence that alcohol consumption rose after Prohibition ended argues against the
belief that drug use would remain constant following the repeal of anti-drug laws.

Page 15 of 64

82. According to the passage, which of the following is most likely to be true about the
relationship between the enforcement of drug-prohibition laws and the street prices of
illicit drugs?
Solution
A) The more strict the enforcement, the higher the street price.
The relationship between the enforcement of drug laws and the street price of
drugs is assumed in the passage to accord with the economic law of supply and
demand. The reason that many drug-control efforts are . . . counter-productive is
Guess
implicitly that they produce shortages or increase the dangers and costs to the
suppliers, thus driving up prices. As a result, many cocaine and heroin addicts
spend hundreds and even thousands of dollars a week. The sentence beginning
If the drugs to which they are addicted were significantly cheaper which would
be the case if they were legalized . . . repeats the theme of a positive correlation
between street price and law enforcement.
B) The more strict the enforcement, the lower the street price.
The passage provides no reason to suppose that the predictable outcome of strict
enforcement, a reduction in the quantity of drugs available on the street, also
reduces their price. (If it did, there would be no particular temptation to corruption
by those engaged in the enforcement.)
C) The less strict the enforcement, the higher the street price.
Enforcement would decline to zero with legalization, the case in which drugs
would be significantly cheaper.
D) There is no systematic connection between the strictness of enforcement and the
street price.
The denial of a connection between the enforcement of drug laws and the price of
drugs contradicts the passage argument (see rationale A).
83. The Bella Coola Indian legend best illustrates the author's point that:
A) humans know very little about the wolf.
To know very little refers to a negative state the absence of knowledge and a
Solution
negative state cannot be illustrated. Furthermore, the prevalence of this negative
state among humans as a whole could not be indicated by a single example but
only by reference to a range of incompatible opinions, which is the authors
method.
Guess
B) wolves provoke fanciful thinking.
The legend recounts the origin of the wolfs human-like stare. The author refers to
it to illustrate the powerful influence on the human imagination exerted by the
wolf.
Page 16 of 64

C) people have always revered the wolf.


The legend does not suggest reverence for the wolf (the origin of which it ascribes
to a failed project), nor can the attitude it indicates be generalized across time and
cultures.
D) wolves resemble humans in certain ways.
The legend itself concerns the resemblance between the direct gaze of wolves and
the human stare. The authors point, however, is not that wolves resemble humans
in any way but that because of the boldness of their regard, they influence humans
to explain the feelings that come over them when confronted with that stare and
thus to create such legends.

84. In the context of the passage, the word rigorous means:


A) unyielding.
While clouds are clearly yielding rather than unyielding, the yielding concept has
Solution
no such relevance to the passage statement about wolves. The author is not
advising readers to yield more to the opinions others express about wolves but to
be more cautious in forming any conclusion.
Guess
B) harsh.
In expressing the futility of attempting rigor in characterizing wolves, the author is
not advising readers to be less harsh in their judgment but to withhold judgment,
since we know little about the wolf.
C) precise.
You might as well expect rigor of clouds indicates the impossibility of being
precise in characterizing wolves. The one-sentence paragraph in which rigorous
appears thus summarizes the idea expressed in the previous paragraphthat
everything we have been told . . . should have been said . . . with the preface that
it is only a perception in a particular set of circumstances.
D) judgmental.
Although the passage does advocate a less judgmental view of wolves, the context
in which rigorous is used you might as well expect rigor of clouds indicates
that its intended sense is the antithesis of a quality to be expected of clouds. A lack
of judgment is hardly a salient quality of clouds.
Page 17 of 64

85. Which of the following statements, if true, would most WEAKEN the author's
contention that the wolf is less known than created by us?
A) The incident the author described as a wolf's "winging a piece of caribou hide
Solution
around like a Frisbee" never in fact occurred.
If even an instance of wolf behavior reported by the passage author were
imaginary, the conclusion that the wolf is a creature of human imagination would
be strengthened.
Guess
B) Nunamiut Eskimos have very little interaction with wolves, and base their beliefs
about wolves on folklore.
If unreliable ideas about wolves were held by even a group living in close
proximity to them, the authors premise would be strengthened.
C) The apparently intelligent behaviors that wolves exhibit, and that people have
always found so intriguing, are entirely instinctive.
The issue of whether wolf behaviors indicate intelligence or instinct is irrelevant to
the issue of whether these behaviors are generally known to humans.
D) Scientists have produced a wealth of knowledge about the wolf, the preponderance
of which corresponds closely to the beliefs of Eskimo and Indian cultures.
If the descriptions of wolf activities provided by Native American observers were
confirmed by a large body of scientifically gathered reports, the grounds for
skepticism about our knowledge of the wolf would be undermined.

86. In organizing a party of natural historians to study wolves, the author would most
likely advise them to approach the wolf with:
A) love, because contrary to popular belief, a healthy wolf will seldom harm a human
Solution
being.
Wolf-lovers want to say no healthy wolf ever killed anyone in North America,
which isnt true. Furthermore, the author would be unlikely to be so
presumptuous as to advise a group of scientists to love the subject of their studies.
Guess

Page 18 of 64

B) caution, because wolves are hunters, and some have been known to kill humans.
The Nunamiut Eskimo of the central Brooks Range speak of wolves as hunters,
and wolves have killed Indians and Eskimos, so the author would be likely to
advise caution in approaching them.
C) contempt, because while they may be similar to domestic dogs, wolves are usually
quite dangerous.
To characterize wolves as quite dangerous would be an exaggeration. Only
wolf-haters want to say they are born killers, which isnt true. Furthermore, the
feelings of people in the presence of wolves includes respect, and there is nothing
in the tone of the passage to suggest contempt for the subject of its authors
investigations.
D) confidence, because we now know much more about the wolf than we did in the
past.
The truth is we know little about the wolf, and the natural historians would be
making their expedition in order to learn more.

87. The 1928 edition of the World Book Encyclopedia defines the wolf as "a rapacious,
flesh-eating animal belonging to the dog family" and states that the wolf "lacks [the
dog's] courage and loyalty." If the author were to include this description in the
Solution
passage, it would probably be used to:
A) support the point that wolves are excellent hunters.
The quotation does not mention hunting. It might have been applied to a
scavenger.
Guess
B) illustrate the point that people often see wolves as being like themselves.
Normal persons would be unlikely to describe themselves in this way.
C) emphasize that what we think we know about wolves is often the product of our
imagination.
The emotionally toned anthropomorphism of the word rapacious and the
disparaging comparison with the domesticated dog in terms of courage and loyalty
(despite the acknowledgment that the wolf belongs to the dog family) indicate
that the writer of the encyclopedia definition relied at least as much on
preconceptions as on objective knowledge.

Page 19 of 64

D) explain the author's own opinion about wolves.


The authors description is far from suggesting rapacity, cowardice, or a lack of
loyalty: Wolves share food with the old who do not hunt, and give gifts to each
other, implying both generosity and loyalty. The fact that they hunt caribou, a
much larger animal than themselves, implies courage. And they spend a good
deal of time with their young, another indication of loyalty.

93. According to the passage, the internal heat of earth is considered a result of all of the
following factors EXCEPT:
A) fluid motions in the outer core.
Solution
The movement of the molten outer part of Earths core is not mentioned as
contributing to its heat. The passage states only that this layer is hundreds of
degrees cooler than the inner core.
Guess
B) primordial gravitational processes.
The gravitational pressure that caused orbiting gas and dust to coalesce into the
primordial planet released enough energy to melt these materials.
C) decay of trapped radioactive elements.
The decay of radioactive elements trapped in Earths interior generates heat.
D) primordial separation of heavier from lighter materials.
The separation of the heavier metallic materials that were to form Earths center
from the lighter silicate minerals that were to form its outer layers generated
energy that added to the internal heat of the planet.

94. According to the information given in the passage, the amount of heat provided by
the sun, in terawatts, is roughly:
A) 42
Solution
B) 4,200
C) 42,000
The passage estimates the 42 terawatts released from Earths surface as a
Guess
thousandth of the heat provided by the sun. The heat from the sun is therefore a
thousand multiplied by 42, or 42,000 terawatts.

Page 20 of 64

D) 42 trillion

95. According to the passage, earth's core is predominantly:


A) solid and radioactive.
B) gaseous and magnetic.
Solution
C) fluid and metallic.
The passage states that the inner third of Earths core is now solid, implying that
the core remains predominantly molten and also mentions fluid motions in the
Guess
outer core. It refers to Earths metallic core? and to the cores being made up
mostly of iron and perhaps some nickel.
D) solid and magnetic.

96. According to the passage, the continuous existence of earth's magnetic field is best
shown by analysis of the:
A) geological activity at the earth's surface.
Solution
Earths surface geological activity and its magnetic field are attributed to separate
sources of heat from within the planet; the passage does not relate the two
phenomena.
Guess
B) magnetic data obtained from rocks.
Magnetized rocks show that the earth has had a magnetic field for at least three
billion years.
C) electrical conductivity of rocks from the earth's mantle.
In order to sustain Earths magnetic field, electrical conductivity must characterize
its core, not its mantle: All these interactions must be working together smoothly
in the outer core, for otherwise the dynamo would have long since ceased to
function.
D) conversion of mechanical into electrical energy by rocks.
The energy conversion is explained as a dynamo effect, which is created by
moving a conductive material through a magnetic field. That is, it is fluid
motions in the outer core, not the material in motion, that causes the conversion,
and this material is the molten iron alloy of the core, not rocks.

Page 21 of 64

97. According to the passage, the dynamo theory derives much of its credibility from the
fact that it alone can account for the:
A) periodic reversals of the earth's persisting magnetic field.
Solution
The only theory that can explain the persistence of the field and its propensity for
reversing itself is the dynamo theory.
Guess

B) weakness of the earth's magnetic field.


Earths magnetic field is not characterized as weak.
C) intense heat of the earth's core.
The core heat is attributed to the planets fiery beginning and to the release of
energy as the heavier materials separated from the lighter ones, differentiating
the core and the mantle.
D) electrical conductivity of the earth's inner core.
The conductivity of the solid inner core is irrelevant to the dynamo theory, which
depends on fluid motions in the outer core.

98. According to the passage, magnetic fields are primarily by-products of:
A) fluid motion.
B) the rotation of the earth.
Solution
C) mechanical energy.
D) electrical currents.
The earths magnetic field, like any other, is a by-product of electric currents.
Guess

Solution

Page 22 of 64

99. The synthesis of geology, p


and other disciplines requir
the one described would be
A) historical and contempo

Guess

artistic inspiration.
Painting styles are varia
separate areas of study,
subjective experience, n
theory.

B) the art, history, and tech


theories about ancient ci
The synthesis of knowle
and mathematics in a the
application of a broad un
forces to an explanation
therefore resembles the c
antiquity in the developm
civilizations.

C) child, adolescent, and ad


develop a theory of intel
The study of possible ag
problem solving does no
disciplines.

D) the effects of different a


pain.
A comparison of the eff
in performing a single fu
to synthesize separate ar

100. The central thesis of the passage is that:


A) we are basically unaware of our conceptual system.
The automatic operation of the conceptual system is mentioned
Solution
as the reason that it must be studied indirectly, through language,
but our lack of awareness of its functioning is not the subject of
the passage.
Guess
B) a culture can view argument as an aesthetically pleasing dance or
as war.
The author uses argument as an example of a concept that is
structured by a metaphor, developing the argument that those
who view argument as war would not recognize the concept as
Page 23 of 64

used by a culture that views it as dance. This example is


discussed only to serve the argument about the pervasiveness of
metaphor.
C) metaphors control our perceptions, thoughts, and actions.
The thesis of the passage is stated in the first two paragraphs,
elaborated and restated in the next two, and clarified with an
example in the remainder of the passage: Our ordinary
conceptual system, in terms of which we both think and act, is
fundamentally metaphorical.
D) metaphor is a poetic as well as a rhetorical device.
The thesis does not concern the conscious use of metaphor,
poetically or rhetorically: metaphor is pervasive in everyday
life, not just in language but in thought and action.

communication, since their goal is to defeat an opponent


through attack, defense, counterattack, etc..
Solution

Guess

102. Given the claims made in the passage, the expressions "She's
brimming with vim and vigor," "She's overflowing with vitality,"
"He's devoid of energy," and "I don't have any energy left at the
Solution
end of the day" would suggest that:
A) some people have more energy than other people.
The individual differences in energy level that can be inferred
from a comparison of the expressions are irrelevant to passage
Guess
claims, which concern the influence of metaphor on conceptual
thought.
B) most people wish that they had more energy.
The desirability of energy can be inferred from the overt content

Page 24 of 64

of the expressions. It is not evident from the metaphor


underlying the concept and is therefore unrelated to passage
claims.
C) many people think of vitality as a substance.
To characterize energy in the same terms as liquid in a container
(brimming, overflowing) or as a quantifiable possession (devoid
of, dont have any) indicates that it is conceptualized as a
substance.
D) some people think that vitality affect our ability to argue.
Neither the expressions nor the passage suggests a connection
between vitality and argumentativeness.

103. According to the passage, if a speaker says, "I've never won an


argument with him," he or she is most likely thinking that:
A) arguments are violent.
Solution
One may win something without having engaged in violence.

Guess

B) arguments are like contests.


The speakers implicit belief that arguments are won by one of
the participants (rather than settled by both) equates arguments
with contests.
C) conceptual systems are metaphorical.
The use of the word win indicates that the speakers concept of
argument is controlled by the metaphor of a competition or
conflict, not that the speaker believes conceptual systems to be
metaphorical.
D) competition is unpleasant.
The statement does not reveal an emotional response to
competitiononly the speakers assumption that arguments, like
competitions, have winners and losers.

Page 25 of 64

104. The ideas discussed in this passage would likely be of most use to:
A) an ambassador to a different culture.
In suggesting that the metaphors underlying abstract concepts
Solution
may differ with culture, the passage implies a need for
sensitivity in presenting and in interpreting ideas crossculturally, as an ambassador must do.
Guess
B) a senator engaged in a serious debate.
A serious debate engaged in by a senator would probably
concern a dispute over legislation or an election, issues that are
concerned with overcoming political opposition and for which
an aggressive orientation (argument is war) may thus be
appropriate. Furthermore, the debate would presumably be
conducted within a culture, so basic conceptual
misunderstandings would be unlikely. Therefore, attention to the
implicit metaphors used in the debate would not serve the
senator.
C) a financial analyst for a large corporation.
The financial analyst would be concerned more with business
and economic data that might affect the corporationverifiable
factsthan with concepts likely to be affected by metaphoric
thinking.
D) a general preparing for battle.
The general would be preparing for a real battle, not a
metaphorical one, a situation to which the philosophical and
psychological ideas of the passage would have little
applicability.

105. An appropriate theory of metaphor derived from the information


contained in the passage would state that metaphor involves:
Solution

I.
II.
III.

understanding one thing in terms of another.


experiencing one thing in terms of another.
communicating about one thing in terms of another.

Page 26 of 64

Guess

A) I only
B) II only
C) I and II only
D) I, II, and III
The idea that metaphor permeates all three aspects of conceptual
functioning is stressed throughout the passage e.g., metaphor
is pervasive in everyday life, not just in language but in thought
and action. Our ordinary conceptual system, in terms of which
we both think and act, is fundamentally metaphorical. The three
ways in which metaphor influences thought are also explicitly
stated in reference to the hypothetical case of a culture with a
dance metaphor for the argument concept: In such a culture,
people would view arguments differently [option I], experience
them differently [option II], carry them out differently, and talk
about them differently [option III].

106. The expression "This is driving me around the bend" would best
support a metaphor that compares madness to:
A) a location.
Solution
The pronoun this refers to a source of stress that has the power
to drive one to madness, which is conceptualized as a location
that is around the bend.
Guess
B) a vehicle.
A vehicle is not invoked; one is being driven in the sense of
being impelled onward before some force. And one is driven
toward madness, not by madness.
C) a road.
Although the concept of a bend suggests a road, it is a road that
one is on before reaching the bend around which lies madness,
so madness is not the road itself.
D) a force.
The driving force is not madness but the stressful circumstance
referred to as this.

Page 27 of 64

107. According to information in the passage, the best way to conduct


an argument with someone from a culture that views an argument
as a dance would be to:
Solution
A) methodically attack the person's weaker claims.
The idea that some of the ideas expressed in an argument are
weaker than others implies a war metaphor. In exchanging
ideas with someone who considers argument a dance for two, an
Guess
appropriate response to claims that seemed tentative or
incompletely supported might be to elaborate them, to ask about
their implications, to consider their usefulness, etc., but not to
attack them.
B) calmly respond to the person's charges.
Since the charge would not be intended as an attack but as a
movement in an interactive performance, the appropriate
response would be considered and graceful.
C) explain that arguments are not like dances.
Since our conceptual system is not something we are normally
aware of, one is not apt to notice the metaphor controlling his
or her own style of arguing and so would be unlikely to identify
that of another culture. We would probably not view them as
arguing at all.
D) irrationally defend your position.
Defense is unnecessary when there is no attack. Furthermore,
irrationality would be unlikely to contribute to a performance
that was balanced and aesthetically pleasing.

Solution

108. Which of the following statements is NOT presented as evidence


for the metaphorical conceptual system on which arguments are
supposedly based?
A) We gain and lose ground.
We gain and lose ground.

Page 28 of 64

Guess

B) We attack an opponent's position.


We see the person we are arguing with as an opponent. We
attack his positions.
C) We plan and use strategies.
We plan and use strategies.
D) We claim that criticisms are on target.
The passage does not cite a metaphoric use of the term on
target.

109. According to the passage, many people think that they can get
along very well without metaphor because they believe that
metaphor is:
Solution

Guess

I.
II.
III.

used only by rhetoricians and poets.


a characteristic of language but not of thought.
a device that alters one's perception of reality.

A) II only
Option II is true but incomplete.
B) III only
Option III: Our concepts structure what we perceive, and our
conceptual system is largely metaphorical. These assertions
express the authors belief that metaphor alters perception. They
do not imply that this effect is widely understood or is believed
to constitute a reason for considering metaphor expendable.
C) I and II only
Option I: Metaphor is for most people a device of the poetic
imagination and the rhetorical flourish. Option II: Metaphor is
typically viewed as a characteristic of language alone, a matter
of words rather than thought. For these reasons, most people
think they can get along perfectly well without metaphor.

Page 29 of 64

D) I, II, and III


See rationale B.

Page 30 of 64

Solution

110. The author's central thesis is that:


A) promises, commitments, excuses, pleas, and pacts are verbal conventions.
Not only are promises, commitments, pleas, and pacts not the focus of the
passage; the passage author implicitly deplores the degeneration of such
performative utterances from sacred rituals into empty verbal conventions:
these . . . are ceremonies or they are nothing.

Guess
B) li is effective in controlling the behavior of others.
The effectiveness of li in directing behavior (inducing a student to bring one
a book) is mentioned only as an example of its incidental power to
accomplish physical ends; it can also be used in this way. Furthermore,
the willful control of others would be antithetical to the concept of li
presented.
C) performative utterances are more honest than are stated intentions.
The honesty of performative utterances is not at issue. They are discussed
only to provide Western examples of the Eastern idea of binding,
ceremonious words.
D) the powers that are uniquely human are revealed in traditional ceremonies.
The thesis topic is the truly, distinctively human powers considered by
Confucius. The central thesis about these powers is that they must be
approached by way of the notion of li, the meaning of which is close to
holy ritual, sacred ceremony. This thesis is elaborated throughout the
passage.

111. The passage suggests that Confucius would be most likely to endorse which of the
following reasons to use li?
A) It acknowledges the dignity of others.
Solution
The idea that unlike coercion, which violates the dignity of others, li works
through spontaneous coordination rooted in reverent dignity is traced to
Confucius.
Guess
B) It works invisibly through magical powers.
The powers that are distinctively human have a magical quality. Implicitly, these
powers are the invisible and intangible forces of li: Through ceremonious
politeness, one may experience the magic of having ones wishes met. That is, li
Page 31 of 64

is magical figuratively in its results, not literally in its means. It is rooted in


reverent dignity, not in the occult.
C) It does not require the expenditure of effort.
The idea that the practice of li does not involve effort is presented as an aspect of
its magical quality, not as a reason for its use.
D) It avoids the risks involved in using force.
The passage does not suggest that risk avoidance is a valid reason to strive for the
perfection in Holy Rite.

112. Which of the following assertions does the author support with an example?
Solution

Guess

I.
II.
III.

Some human powers have a magical quality.


Words can function as actions.
Much of human existence consists of ceremony.

A) I only
Option I is true but incomplete.
B) III only
Option III is asserted, but much of human existence is so broad and vague a
concept that no example of it could be offered.
C) I and II only
Option I: A book from my office that appears in my hands, as I wished is an
example supporting the assertion that some human powers have a magical quality.
Option II: I give and bequeath my watch to my brother is an example supporting
the assertion that words can function as actions.
D) II and III only
See rationale B.

Page 32 of 64

113. The powers discussed by Confucius are described as magical. What is the most
likely reason for the choice of this word?
A) The word directs attention to the power behind ordinary social conventions.
Solution
The adjective magical refers to powers so familiar and universal as to be
unnoticed. This context, as well as that in which the noun magic later appears,
expresses a sense of wonder at the effectiveness of conventional social forms: The
only action taken is the proper ritual expression of my wish, yet the wish is
Guess
accomplished without effort on my part. The magical quality of such
transactions is emphasized by the phrases merely express ... my wish and in
almost no time as well as by an exclamation point.
B) The rituals involved in li harness supernatural phenomena.
There is no suggestion that the revelations of Confucius concerned supernatural
methods. The dimension of human existence to which the word magical is applied
is unnoticed only because it is already so familiar and universal.
C) These powers can be learned only from Confucian masters of the occult.
The occult is not invoked, and although one has to labor long and hard to learn
li, there is no suggestion that the services of a master are required. The difficulty
in acquiring the ability to act according to li and the will to submit to li is not
that the concept is difficult to understand but that its perfection is esthetic as well
as spiritual.
D) Those with these powers can move physical objects through the use of words
alone.
The moving of objects by means of words is presented as an incidental aspect of
li: It can also be used to accomplish physical ends, although we usually do not
think of it this way. Furthermore, in the example of a book that appears without
being actively sought by the author, the words do not act on the desired object but
on one of my students, who willingly delivers it.

114. According to the passage, which of the following sentences is/are performative
utterances?
Solution

I.
II.
III.

Please help me.


I promise to help.
May I help you?

Guess

Page 33 of 64

A) I only
A request (option I) does not constitute the performance of an act by the speaker.
Rather, it is a stimulus to action by another.
B) II only
Option II is a performative utterance because to utter the words I promise is to
commit oneself through ones words. The utterance is the very execution of the
act [of commitment] itself, in the same way as I give and bequeath my watch .
. . is not a report of what I have already done but is the very act of bequeathal.
C) I and III only
A question (option III) does not constitute the performance of an act by the
speaker. Rather, it is a stimulus to action by anotherthat of replying.
D) II and III only
See rationale C.

115. The author suggests that Confucius considered the ceremonies of society to be
holy because they are:
A) expressions of respect.
Solution
The adjective holy refers to ritual, the concept in which the Confucian term li
originated, indicating that Confucius revered tradition. In the discussion of the
teachings of Confucius, the description of conventions as authentic and
reasonable, the association of li with reverent dignity, and the contrast drawn
Guess
between the user of li and one who uses coercive means further imply that for
Confucius, the holiness of ceremonies derived from the respect for self, for others,
and for tradition that they express.
B) divinely sanctioned laws.
The passage does not suggest that Confucius believed in the existence of a divine
lawgiver capable of sanctioning social conventions.
C) customary in refined society.
The ceremonies extolled by Confucius conveyed distinctively human powers.
They were not the ephemeral customs of a privileged class but a familiar and
universal . . . dimension of [human] existence.

Page 34 of 64

D) performed by holy persons.


Although one has to labor long and hard to learn li, and although without li, one
cannot possess that perfect and peculiarly human virtue or power, there is no
suggestion that only holy persons succeed in learning to perform social
ceremonies or that persons who practice holy rites become holy themselves.

116. What does Austin's concept of performative utterances imply about language?
A) Language has no direct relationship to actions.
Austins idea is that language sometimes is the action to which it refers.
Solution

Guess

B) Language is sometimes used to conceal facts.


Although performative utterances may sometimes be intended to conceal facts
e.g., a false declaration made under oaththis use of language is not implicit in the
concept.
C) Language and actions can be inseparable.
Austins idea, that innumerable statements . . . are the very execution of the act
itself means that in declarations in which the verb refers to ones own current
linguist act (I admit . . ., I assure you . . ., I name you . . .), language is inseparable
from the action being taken.
D) Language is necessary for social conventions.
The idea that social conventions involve ritual words (but not that they require
them) is implicit in the passage. However, this idea is not implied by the concept
of performative utterances, which refers to particular forms of ceremonial speech,
not to social conventions in general.

117. The author argues that people bind themselves more inescapably through words or
ceremony than through strategies or force. Which of the following claims, if true,
would most WEAKEN the argument?
Solution
A) People rarely try to meet their moral obligations.
Moral obligations are often expressed socially by ceremonial vows and promises.
A general tendency to violate moral obligations would therefore directly

Page 35 of 64

Guess

contravene the argument that ones own words and the ceremonies in which one
participates are more binding on ones behavior than are external constraints.
B) People rarely act ceremoniously.
A rarity of ceremonious actions would not indicate that they are not considered
binding. On the contrary it would suggest an appreciation of their seriousness.
C) People often respond to force or strategies.
The demonstrated power of external forces to compel behavior is irrelevant to the
argument that the force of ceremoniously undertaken obligations is even stronger.
D) People often avoid making promises.
A disinclination to make promises would not indicate that such ceremonious acts
are not considered binding (see response B).

118. According to the author's account, a Confucian would be likely to place a high value
on a person who is:
A) gifted at creating magical illusions.
Solution
The forces at work in li are invisible and intangible, but they are also familiar
and universal in humans, not illusory, and there is no suggestion that they create
illusions. The idea of giftedness also seems antithetical to Confucian thought, not
only because of the universality of the powers it concerns but because of the
Guess
difficulty of perfecting them through li.
B) sensitive to the feelings evoked by ceremonial acts.
The author suggests that a Confucian would value those who show consideration
for others by equating the ceremonious manner and ritual speech of li with
politeness: I turn politely, i.e., ceremonially . . . [and] express in an appropriate
and polite (ritual) formula my wish.
C) clever in applying techniques of verbal persuasion.
Cleverness in the manipulation of others implies a lack of the reverence and
spiritual qualities that characterize li. The term verbal techniques also trivializes
the Confucian concept of sacred forces.
D) talented at teaching philosophical abstractions.
Page 36 of 64

The Confucian teachings did not concern philosophical abstractions but the entire
body of the authentic tradition and reasonable conventions of society, and the
passage suggests that a Confucian would value a practitioner of li more highly
than a teacher of abstract ideas. Furthermore, the word talented implies inherent
ability, whereas one has to labor long and hard to learn li.

119. People often use conventionally polite words and behavior toward those whom
they do not respect. What question might this information reasonably suggest
about the author's explanation of li?
Solution
A) Whether the forces at work in li are necessarily invisible
Whether a disrespectful attitude can be detected beneath superficial politeness is
irrelevant to the invisible forces at work in li, which are rooted in reverent
dignity.
Guess
B) Whether li might not be just cynical manipulation
See rationale A.
C) Whether the English words ritual and ceremony adequately convey the
reverence of li
The fact that in our culture, the conventions of politeness often mask an
underlying lack of respect suggests that the spiritual quality of the Confucian
teachings about traditional forms is not adequately conveyed by such English
nouns as ritual and ceremony. This consideration probably influenced the author
to modify these nouns with the adjectives holy and sacred when using them in
reference to li.
D) Whether li might express contempt as well as respect
See rationale A.
120. The author's claim that some popular
songwriters can be "dedicated, energetic
poets" is supported by:
Solutio
A) the testimony of some literature teachers.
n
Although some teachers of literature are
exempted from the general indifference
that the passage author considers
Guess
common, no testimony from them is
cited.

Page 37 of 64

B) examples of popular song lyrics.


No example of song lyrics is provided.
C) comparison with more academic poets.
These song writers are more interesting
than many of the . . . randy academics.
D) an analysis of Coleridge's poetry.
The name Coleridge is simply mentioned
as an item on a curriculum.

121. The author most likely believes that one of the main purposes of teaching fiction and
poetry should be to provide students with:
A) an awareness of major literary themes.
Solution
The author writes disparagingly and rather sarcastically of this restriction in
literary instruction: since the life-enhancing value of a course in literature is
difficult to measuresince, moreover, many people in a position to put pressure on
educational programs have no real experience in or feeling for the artsit is often
Guess
tempting to treat life-enhancement courses as courses in useful information . . .
[so] books are taught . . . because, as a curriculum committee might put it, they
illustrate major themes in American literature.
B) a sense of objectivity.
Putting a course in literature on the same objective level as courses in civics,
geometry, or elementary physics is attributed to ignorance.
C) an ability to analyze texts.
Although the passage author acknowledges that the message of anthologies
designed to teach analysis is in important ways true, the passage opposes the
classroom emphasis on form: the whole idea of the close critical analysis of
literary works has had the accidental side effect of leading to the notion that the
chief virtue of good poetry and fiction is instructional.
D) engaging experiences.
The passage author evidently believes that the business of education is to give the
student life-enhancing experience, the incomparably rich experience we ask and
expect of all true art, experiences that can delight and enliven the soul.
Page 38 of 64

122. Given the information in the passage, if "drugstore fiction" were taught in literature
classes, which of the following outcomes would most likely occur?
A) Teachers would realize that the interpretation of literature varies depending on
Solution
students' reading skills.
The concepts of various interpretations and differential reading skills are not
addressed.
Guess
B) Committees would change curriculum objectives for most literature courses.
The passage author contends that in planning the curricula of literature courses,
academic committees choose books that illustrate major themes in American
literature, or present a clearly stated point of view. The passage does not
suggest that these objectives would change if the curricula included popular
fiction.
C) Students would enjoy the classes more.
Drugstore paperbacks are cited as an example of the kinds of poetry and fiction
that go most directly to those values we associate with simple entertainment.
D) Students would better understand literary terms like "point of view."
Since the analytic emphasis in literature classes is attributed to people who have
no real experience in or feeling for the arts, the inclusion of popular fiction in
literary courses would probably not affect this emphasis. Nevertheless, the passage
provides no reason to conclude that textual analysis would be better served by the
popular works than by traditional course materials.

123. The author suggests that the study of literature is much less rewarding when it:
A) focuses solely on critical analysis.
According to the passage author, the rewarding aspect of works of literature is that
Solution
they give joy, the incomparably rich experience we ask and expect of all true art.
This experience occurs during reading; it is immediate. It is therefore
presumably weakened during a careful analysis of the text for its exploration of
ideas [and] . . . development of the implications of a theme.
Guess

Page 39 of 64

B) does not provide students with useful information.


The passage author opposes the treatment of life-enhancement courses as courses
in useful information and extols reading for pleasure as the rewarding experience
offered by literature. This position implies that a disregard for useful information
would be more likely to enhance than to reduce the rewards of literary study.
C) concentrates on the pleasures of merely commercial entertainments.
While acknowledging the boring sameness found at its extreme in the scripts of
television Westerns, cop-shows, and situation comedies . . . [which are] merely
commercial, the passage author does not condemn all commercial entertainment
nor denigrate the pleasure it can provide, arguing that popular song writers can be
. . . more interesting than many [contemporary 'serious writers] . . . and that
drugstore fiction can often have more to offer than fiction thought to be of a higher
class.
D) is treated as a life-enhancing experience.
The life-enhancing value of courses in literature is asserted. The problem
identified by the passage author is that this value is misunderstood by insensitive
but influential persons and so supplanted by inappropriate criteria that are more
easily measured.

124. Suppose the majority of high-school students have read at least one work by the
science-fiction writer Isaac Asimov for a literature class. This new information
would most CHALLENGE the claim that:
Solution
A) no piece of fiction can be lasting if it is simpleminded.
They would presumably not be simpleminded but rather among the drugstore
fiction that has more to offer than fiction thought to be of a higher class.
Furthermore, the information provided does not imply that the works chosen had
Guess
already proved to have lasting merit.
B) literature courses usually meet prescribed curriculum objectives.
No information is offered about over-all course content. However, even if the
classes deal exclusively with popular works, the passage argument does not justify
the conclusion that supposedly analytic curriculum objectives have been
jeopardized. The works students read and the success of teachers in meeting their
instructional goals are separate issues.

Page 40 of 64

C) the people who determine school curricula are generally ignorant of literature.
The reason for the curriculum choices is not apparent. The inclusion of works by
Asimov might indicate either an appreciation of their originality and interest or the
ignorance of those with no real experience in or feeling for the arts.
D) literature teachers are prejudiced against popular genres.
The passage implies that teachers of literature have some latitude in choosing the
works covered in their classes. Therefore, the information that the science fiction
was read for high-school classes suggests that many teachers of literature are not
prejudiced against popular genres.
125. The passage suggests that the most favorable portrayal that Euripides gave of the
Olympian gods was to interpret them as:
A) psychological forces.
Solution
In a period when obvious attacks on the Olympian gods might be severely
punished, a writer would have been foolish to denigrate them consistently. The
passage author indicates that Euripidess portrayal of the gods (as in the
Bacchae) as profound psychological forces was relatively favorable by
Guess
contrasting it with his treatment of them elsewhere as shady seducers or
discredited figures of fun.
B) Divine Saviours.
The divine saviors appeared in the Hellenistic age, which followed the time of
Euripides.
C) inquisitive spirits.
The adjective inquisitive refers to the spirit of Euripides, not to the gods.
D) figures of fun.
The negative tone of figures of fun is indicated both by the introductory phrase
was capable of presenting them as and by the adjective discredited.

126. Suppose that Socrates had said to his prosecutors, "I will obey my god rather than
you." How would this information affect the claim that the prosecutors were correct
to accuse Socrates of not believing in the city gods?
Solution
A) It would refute the claim.
See rationale B.

Page 41 of 64

Guess

B) It would support the claim.


The unelaborated designation my god would permit no inference about the
orthodoxy of Socrates. Although the possessive pronoun my suggests the concept
of a personal god rather than one worshipped by an entire city, and the singular
noun god implies the exclusion of some of these gods, the statement may indicate
only a preference for a particular god in the city pantheon. These points would
have to be resolved to determine whether the prosecutors were correct in their
accusation.
C) It would support the claim if it could be shown that the god to whom Socrates
referred was an Olympian figure.
Since the Olympian powers implicitly included the gods in whom the city
believes, the allegiance of Socrates to one of these powers would support the
accusation that he did not believe in [any] of the city gods only if the god in
question were not also specifically associated with the city.
D) It would support the claim if it could be shown that the god to whom Socrates
referred was not an Olympian figure.
Since the Olympian powers implicitly included the gods in whom the city
believes, the demonstration that the god obeyed by Socrates was not among these
powers would confirm the accusation that he did not believe in the city gods.

127. Suppose it is discovered that a Stoic philosopher originated the traditional portrayal
of the Olympian god Zeus as the source of thunder. How would this information
affect the author's claims about the Stoics?
Solution
A) It would support the claim that the Stoics reinterpreted individual deities as
explanations of natural phenomena.
The discovery that Zeus was first portrayed as the source of thunder by a Stoic
would imply that the identification of a god with a force of nature accorded with
Guess
the Stoic philosophy, supporting the passage assertion that Stoics reinterpreted
and accommodated many individual deities as merely allegorical explanations of
natural phenomena.
B) It would support the claim that the Stoics came to regard the Olympian gods as
merely symbolic.
Although they reinterpreted and accommodated many individual deities as
merely allegorical, the Stoics maintained their belief in divine Providence and
Page 42 of 64

are therefore contrasted, in the phrase even the Stoics, with those who regarded
all Olympian gods as merely symbolic.
C) It would weaken the claim that the Stoics believed in divine Providence.
The idea that Zeus is the source of thunder can be interpreted literally (as a humanlike god hurling thunderbolts) as well as allegorically (as a personification of the
godlike power of thunder). The former interpretation is clearly consistent with a
belief in divine Providence, and the latter is not necessarily inconsistent with it.
D) It would weaken the claim that the Stoics produced numerous slighting references
to the Olympian gods.
These references are not attributed to the Stoics but to an extended period of time:
the early Hellenistic age that followed produced numerous slighting references to
the Olympian powers.

128. The claim that religion was one of the most vital elements in the Hellenistic world is
based mainly on the:
A) comments of St. Paul regarding pagan Hellenism.
Solution
This assessment was that the Hellenistic religion was worthless: St. Paul . . . saw
pagan Hellenism as a world without hope and without God.
Guess

B) existence of cults devoted to Divine Saviours.


This assertion is a conclusion based on the discussion in the final paragraph of the
emergence of various influential cults devoted to a belief in divine saviors while
the Olympian gods continued to receive ceremonial worship.
C) writings of Menander and Epicurus.
Menander and Epicurus both wrote that the traditional city gods, if they existed,
were indifferent to human misery. This conclusion implies that religion was not a
vital element in the Hellenistic world.
D) idea that Olympian gods had once been human kings.
The idea that the gods had descended from humans was: little more than a
rationalization of atheism.

Page 43 of 64

129. The Iliad and the Homeric Hymns are cited in the passage as evidence for the claim
that:
A) the rationalist movements of the fifth century diminished the reputations of the
Solution
Olympian gods.
The two works are not identified with the rationalist movements. Furthermore,
these movements are dated in the later fifth century B.C. and the adverb further
indicates that this period followed the age in which the two works were produced.
Guess
B) poets believed that the old gods were no longer a matter of serious concern.
Homer is not explicitly identified as a poet. However, even if this knowledge is
assumed, his flippant treatment of the gods in two works is not presented as
indicative of the beliefs of poets in general.
C) Greeks had a tendency to treat the Olympian gods irreverently.
The Greeks is the subject of the first sentence of the passage. The second
sentence implicitly refers to the Greeks in asserting that there had always been a
tendency . . . to treat the gods with a certain familiar flippancy and cites the two
works as clear early evidence of this tendency: this is already very apparent in the
Iliad and the Homeric Hymns.
D) some Greeks believed that the gods of the city were indifferent.
The Athenians hymn proposing that the city gods were indifferent is not
associated with either of the two works. Furthermore, this proposal was made
meanwhile i.e., during the early Hellenistic age that followed [the time of
the rationalist movements, Euripides, and Socrates]. The hymn therefore dates
from a period long after the time of Homer (see rationale A).

130. The passage suggests that, before Callimachus and Theocritus, Greek poets
presented portraits of the Olympian gods that were:
A) similar to sculpture.
Solution
It is the approach of the poets Callimachus and Theocritus, not that of their preHellenistic predecessors, that is likened to that of the sculptors of their time.
Guess

B) flippant and unflattering.


The assertions that there had always been a tendency . . . to treat the gods with a
certain familiar flippancy and that Euripides was capable of presenting them as
Page 44 of 64

shady seducers or discredited figures of fun do not imply that pre-Hellenistic


poetry in general portrayed the gods disrespectfully. If the passage author had
meant to convey this idea, the Hellenistic poets Callimachus and Theocritus would
not have been characterized as living in an age when the old gods were no longer
a matter of belief or serious concern.
C) virtually atheistic.
The Greeks were traditionally a religious people, and the attitude toward the
gods attributed to the earlier Greek poets does not suggest atheism: a certain
familiar flippancy and an assault on their reputation by Euripides, who
nevertheless interpreted them as profound psychological forces.
D) highly idealized.
The comparison of Callimachus and Theocritus to Hellenistic sculptors, who
began to represent some of these gods in much less idealistic forms than those
their predecessors had favoured, implies that the poets who preceded these two
had also idealized the gods. The statement that the two poets were living in an
age when the old gods were no longer a matter of belief or serious concern also
implies that earlier poetry had expressed reverence toward the gods.

131. The passage as a whole suggests that in order for a religion to retain its vitality, it
must:
A) abandon the search for miraculous gifts.
Solution
The passage suggests, on the contrary, that the savior cults supplanted the pagan
gods by promising miraculous gifts that everyone strongly desired.
Guess

B) respond in some way to people's needs.


The idea that religions can thrive only if they respond to human needs is implicit
throughout the passage: Many people had come to regard them as merely
symbolic . . . merely allegorical explanations of natural phenomena; the gods of
the city . . . were at least indifferent; the traditional gods seemed able to do
nothing to ease peoples daily encounters with the vicissitudes of Hellenistic life;
a world without hope . . .; Saviours were relied on, passionately, for . . . the
conferment of strength and holiness to endure our present life . . . [and]
immortality and happiness after death.
C) answer the challenges set forth by rationalists.
Rationalism is not presented as the principal reason for the decline of the
Page 45 of 64

Olympian gods: Pagan religion was not already dying or dead when Christianity
overtook it; it had remained very lively indeed. But it had deviated . . ..
D) be attractive to poets and other artists.
The passage does not imply that Greek paganism was uncongenial to artists in
particular or that they incited devout people against it. It cites the writers and
artists of ancient Greece as recorders of views that were prevalent in the
population: A general tendency to flippancy toward the gods is apparent in poetry;
Euripides presented the gods irreverently on stage at the same time that Socrates
questioned traditions; poetry reflected the beliefs and concerns of an age.
132. The author's main point about television is that it:
A) relies predominantly on series for its programming.
This statement is made only to support the point that such programming gives
Solution
viewers the comfort of familiarity, part of the argument that television viewing
produces passivity and inertia.
Guess

B) portrays life more realistically than Hollywood films do.


Television programs are contrasted with Hollywood films only once, to make the
point that television is a reality factory. That is, television only seems to portray
life realistically (unlike the dreams provided by Hollywood), while manufacturing
a false version of it.
C) uses emotional appeals as part of its news reports.
The passage refers to the viewers emotional response to news reports (not to
deliberate emotional appeals by reporters or victims), and the point of the
reference is that the segmented presentation of the news neither enables viewers to
resolve empathetic feelings nor encourages them to take action. But the main focus
of the passage is not news reporting or the public response to it; it is the synthetic
realism of television as a whole.
D) presents a sanitized version of reality.
The author distinguishes between being true to life, as television is, and life itself,
arguing that the predictable and scripted naturalism of television lacks the
complexity, spontaneity, and passion of real life.

133. The passage suggests that television newscasters attempt to do which of the

Page 46 of 64

Solution

Guess

following to their viewers?


A) Encourage them to stay passive
Implicitly, television newscasters intend to induce passivity, since the reward of
being a viewer depends on staying passiveif we are moved to leave the viewing
chair we may miss the next program. Such movement then is rarely any part of a
newscasters program: stay tuned is the eleventh and commanding
commandment.
B) Help them achieve emotional catharsis
According to the passage, victims or survivors on the screen arouse the viewers
emotions, but catharsis does not follow, implicitly because of the immediate
replacement of one segment by the next in the plethora of news . . . hosing us
with information.
C) Move them to leave the viewing chair
See rationale A.
D) Confuse them by compartmentalizing information
In stating that the segmentation and personalization of the news actually
confuse us with their discontinuous gush, the passage does not suggest that
newscasters deliberately attempt to confuse their viewers.

134. The passage suggests that a real-life argument between two people would probably
not make for good television because the people would be:
A) less convincing than professional television actors.
Solution
The apparent authenticity of televised drama is not attributed to the quality of the
acting but to the careful naturalism of the language: grammar smacks of
premeditation. The suggestion that people model their behavior on that of
television characters, learning to enact the kind of behavior in life which will
Guess
make a good program, also concerns the influence of televisions
depersonalized language and familiar scripts, not the skill of its actors: Television
writes our scripts.
B) incapable of articulating their emotions precisely enough.
According to the author, it is the language of mass communication that lacks
precise articulation because it follows the principle that a lack of articulacy is
the badge of sincerity.

Page 47 of 64

C) more passive than the characters in typical television drama series.


The characterization of television viewers as passive and the suggestion that
television drama sets standards for real-life interpersonal behavior are separate
ideas. The passage does not suggest that the passivity encouraged by television
viewing is expressed in human relationships.
D) too passionate to fit in with the prevailing language of television.
The charge that television gives us back our language . . . docked of amateurish
or embarrassing passions or obsessions which might cause our audience to switch
off suggests that the emotions expressed in a genuine disagreement would be
inconsistent with the conventions of television scripts.

135. The overfamiliarity of certain television characters or advertisements is cited by the


author as evidence for the claim that:
A) there is little place for originality on television.
Solution
The author bases this conclusion directly on the repetitiveness of television:
characters we have seen before, advertisements we love may well be the
evidence that originality (what has never been said before) has scant future on the
box.
Guess
B) news is not the only programming that has a numbing effect on viewers.
The comments about the domination of programming by series and the
immediately following idea, attributed to Malcolm Muggeridge, that the plethora
of news has a numbing effect on viewers, are implicitly related in that both
concern repetition. However, the author does not imply that repetition has a
numbing effect on viewers of nonnews programs.
C) television is truer to life than life is.
According to the passage, it is the naturalism and regular recurrence of television
dramatic programs that give viewers a sense of authenticity. Although the
characters and commercials seen in a series would necessarily become familiar to
viewers, the author does not cite this familiarity as a reason for the conclusion that
television seems more authentic than life itself.
D) television convinces through immediacy and repetition.
No evidence is offered for the assertion that television convinces us by
immediacy and by repetition. The familiarity of the language, plots, characters,
Page 48 of 64

and advertisements of television is mentioned as an aspect of its repetitive nature,


not as evidence that it is convincing.

136. The existence of which of the following phenomena would most strongly
CHALLENGE the information in the passage?
A) A critic who enjoys television drama series
Solution
The author does not suggest that other television critics generally oppose
serialized dramas.
Guess

B) A news show that employs fast-paced images and simple language


A news show characterized by a rapid progression of images and by simple
language would accord with the authors reference to the discontinuous gush of
such programs.
C) A sympathetic television character who speaks passionately and articulately
The presence on television of an appealing character whose speech is both
passionate and articulate would challenge the passage authors contentions that a
lack of articulacy is the badge of sincerity and that television language is docked
of amateurish or embarrassing passions.
D) A television series that mixes and recycles plots from a variety of sources
A series that combined recycled plots from various sources would support the
authors charge that television is a voracious recycler and mixer of a confluence
of concepts.

137. According to the passage, a "good program" would best be described as one that:
A) forces viewers to feel the full impact of their emotions.
Viewers who felt the full impact of their emotions (as opposed to simply
Solution
experiencing a rapidly alternating sequence of unresolved emotional responses)
might be moved to leave the viewing chair to take political or social action.
According to the passage argument, such a consequence would be contrary to
programming goals.
Guess

Page 49 of 64

B) involves viewers just enough to keep them from leaving their chairs.
"A 'good program'" is presumably a rewarding one for the viewer, and the reward
of being a viewer depends on staying passive.
C) features passionate and articulate characters.
According to the author, television does not convince us by structured argument
or oratorical exposition. A good program is one that avoids dealing
intelligently with passionately held ideas.
D) shows interesting and unusual aspects of everyday life.
A preference for what has been said before characters we have seen before,
advertisements we love implies that interesting and unusual aspects of everyday
life would not be considered suitable topics.

Page 50 of 64

Biological Sciences Solutions for AAMC 3R


140. The Regulative Hypothesis proposes that each cell contains complete information for

construction of the multicellular organism. Choice D is just a restating of this hypothesis.


It can not be a correct answer, because the Regulative Hypothesis contradicts the Mosaic
Hypothesis. If a single egg produces identical twins or triplets, identical plans for each
individual must have been transferred from the egg in accordance with the Regulative
Hypothesis, so A is also a wrong answer. For an embryo to have two normal-sized heads,
identical construction information must have been introduced into both parts as stated in
the Regulative Hypothesis, so B is also wrong. The Mosaic Hypothesis, states that the
parts of the plan are handed out to the cells that need them at the time of division. The
production of partial embryos is consistent with the mosaic theory because it implies that
each embryo received just part of the total construction plan. Answer choice C is the only
one of the four statements that supports the Mosaic Hypothesis.
141. Construction information for an organism must be coded in the form of the DNA in the

nucleus of the egg. If the nucleus of an egg cell can be replaced by the nucleus of a gut
cell and still direct the production of a complete individual, then the gut cell must contain
all the construction information an egg nucleus contains. This is an example of the
Regulative Hypothesis. This narrows the choice of answers to either A or B. Answer
choice A suggests that we should decide that the experiment supports the Regulative
Hypothesis because an environmental factor induced the development of the egg, but, in
fact, the way the egg was activated to begin development is irrelevant to the question of
the source of the information guiding development. Only answer choice B points out a
relevant fact: that the egg with a gut nucleus developed into a complete organism.
142. The development of incomplete embryos, described in choice A, falsifies the

Regulative Hypothesis. Answer choice B states that the fate of a cell depends on its own
internal factors, but this would be true whether the Mosaic Hypothesis or the Regulative
Hypothesis was true. Cells must have genes to guide their fate. The question is whether
they have a full complement of the organism's genes or just a part. Answer choice D
restates the Mosaic Hypothesis, that genes are distributed unevenly to daughter cells
during development. Answer choice C, the correct answer, however, supports the
Regulative Hypothesis. If all cells contain identical information for construction of the
organism, then some external factor must tell different parts what they should become. If
specific courses of development were shown to be dependent on the position of the cell in
the embryo, it would explain this aspect of the Regulative Hypothesis. By supplying an
answer to the question of how the parts of an animal develop differently when they have
the same genes, that information buttresses the Regulative Hypothesis. Thus, answer
choice C is the best answer.
143. When the position of the cells was switched, the cells took on a fate consistent with

their new positions. Answer choice A is the correct answer. It would be hard to deny that
there must have been "position-dependent cellular interactions" involved. Answer choice
B can not be true because the switched A cells did not develop the way A cells normally
do, but instead developed the way B cells developed. They took on B cell characteristics,
which would be impossible if they had a different set of genes. Answer choice C is not
Page 51 of 64

relevant to the experiment and is true of development in general. It is a statement of the


obvious: that embryogenesis is a process dependent on the organism's individual cells.
Choice D is contradicted by the experiment. Development did vary when the position of
the cells was switched. There fore, answer choice A is the best answer.
144. After meiosis, human germ cells contain half the genetic material of other cells in the

body, but they still retain one copy of each gene. The normal complement of genes is reestablished at fertilization, before development of the embryo begins. The facts stated in
answer choice A are therefore irrelevant. Answer choice B is not good evidence for the
Mosaic Hypothesis. It just suggests that all genes are not active at all times. The
Regulative Hypothesis fits our understanding of human development best and because
cell position does have an effect on development in humans. The correct answer must be
answer choice D.
145. Answer choice D is the only choice that shows that all cells of the body have the same

genetic makeup, because it is the only one in which the chromosome content at the 64
cell stage matches the chromosome complement at the two cell stage. Answer choices A
and B show a reduction in the number of chromosomes and answer choice D shows an
increase. Thus, answer choice D is the best answer.
146. Data that showed that all embryonic cells have the same developmental potential

(answer A), that interactions occur among cells (answer B) or that the fate of cells
depends on external conditions (answer C), would constitute evidence for the Regulative
Hypothesis not the Mosaic Hypothesis. The only choice that is consistent with the Mosaic
Hypothesis is answer choice D. Evidence that the fate of cells depends on their position
disproves the theory that their fate depends on restrictions in the number of genes the cell
contains. Cells can not acquire features based on their position unless they already have
genes for those features.
147. The passage states that an insoluble form of casein forms as the pH decreases from 6.6

to 4.6. Sour milk is acidic. Thus, answer choice A is the best answer.
148. The passage mentions three specific classes of compounds: (1) Calcium caseinate,

which precipitates at low pH as casein; (2) Proteins (lactalbumins and lactoglobulins),


which precipitate when denatured by heat; and (3) Lactose, which hydrolyzes to
galactose and glucose. In the procedure mentioned in the passage, the powdered calcium
carbonate is probably added to neutralize any remaining acetic acid. Solid 1 (casein)
forms on the addition of acetic acid and is removed by filtration, so it is not present when
the CaCO3 is added. Solid 2 (denatured and precipitated proteinaceous material) is
insoluble in the CaCO3. Thus, answer choice A is the best answer.
149. As seen in the equation, the hydrolysis of lactose results in the net addition of H2O

during the cleavage of the disaccharide. Thus, answer choice B is the best answer.

150. Solid 2 forms after the solution is heated to boiling, a condition that denatures and

precipitates the lactalbumins and lactoglobulins. Thus, answer choice C is the best
answer.

Page 52 of 64

151. Patient A in group 3 received a placebo, and so was untreated. Nevertheless, a response

was observed, a reduction in symptoms (lower rating on the 5-point scale). The correct
answer, therefore, is choice A, that the response was one not associated with treatment.
The other answer choices, B, C and D, would only be plausible if the patient in question
had received treatment with a histamine blocker, an acetylcholine blocker or drug with
both properties. Thus, answer choice A is the best answer.
152. Drug A blocks histamine receptors and acetylcholine receptors. The group that received

Drug A reported the lowest level of symptoms. This suggests that decreases in the effect
of both histamine and acetylcholine should be the optimum treatment. Answer choice D
represents this combination. The histamine component of the symptom reduction is
achieved by blocking its histamine receptors with an antihistamine. Introducing
acetylcholinesterase, an enzyme that catalyzes the degradation of acetylcholine, reduces
the acetylcholine component. The question assumes that the examinee knows that
acetylcholinesterase breaks down acetylcholine or at least recognizes that the term refers
to an enzyme for which acetylcholine is a substrate. Thus, answer choice D is the best
answer.
153. The passage states that drug B blocks histamine receptors only. Group 2 received Drug

B. Of all the subjects in group 2, subject C reported the lowest level of symptoms.
Answer choice D is therefore the correct response: that the person who received the most
benefit from blocking of histamine receptors alone was subject C in group 2.
154. Epithelial cells cover all the free surfaces of the body forming an interface between

body tissues and the external environment. When fluid is released from the body it must
cross an epithelial surface. Answer choice A is therefore the correct choice. The cells that
secrete mucous are primarily found in mucosal glands made up of ectodermal epithelium
continuous with the epithelial lining of the nasal cavities. None of the other types of
tissue named, connective (answer B), contractile (answer C) or neurosecretory (answer
D) impinge on the external environment.
155. Acetylcholinesterase is an enzyme that catalyzes the degradation of the

neurotransmitter acetylcholine into choline and acetic acid. An acetylcholinesterase


inhibitor prevents the breakdown of acetylcholine. This causes acetylcholine to remain in
the synaptic cleft for a longer period of time, stimulating the nasal epithelial cells to
secrete. The correct response to the question is therefore answer choice C, that
acetylcholinesterase inhibitors must increase parasympathetic activity at the acetylcholine
receptors. Answer choice D states the opposite, that it decreases activity. Answer choices
A and B are incorrect because they describe alternatives that would decrease rather than
increase synaptic conduction.
156. The passage states that cholesterol is a precursor of steroid hormones. Of the

compounds listed, estrogen is a steroid hormone. Therefore, the key is D.


157. Evidence that familial hypocholesterolemia (HC) is caused by a genetic disorder comes

from two facts: (1) it is relatively common in some families and absent in others, and (2)

Page 53 of 64

it develops independent of diet. This combination is described in choice A. Answer


choices B and C imply that it is related to diet. Answer choice D suggests that this
consistent familial protein defect might be independent of a DNA defect. Because DNA
codes protein structure and DNA is the molecular basis of heredity, this cannot be a
correct choice. Therefore, answer choice A is the best answer.
158. The cause of HC presented in the passage is the inability of membranes of affected

individuals to bind the blood-protein that transports cholesterol, the low-density lipid
binding protein, LDL. The drug described in answer choice B would be the best model of
HC. Such a drug could, by preventing membrane binding of LDL, disrupt normal
functioning in the same way as proposed for HC. Increasing absorption of lipids by the
small intestine (answer A) would not mimic HC as well; it would just increase all types
of lipids in the blood. Lowering the bile in the small intestine (answer C) would prevent
lipid emulsification, hence absorption, but would not be a good model. Decreasing
plasma membrane protein production (answer D) would not be a good model because it
would produce widespread effects on the body that would be difficult to untangle from
the effect of loss of membrane binding proteins alone. Thus, answer choice B is the best
answer.
159. Both the father and mother have cholesterol levels of 3.0 mg/mL, which is in the

moderately affected range. This suggests they have one copy of the HC gene and one
copy of the normal (wild-type) gene and that the two versions of the gene (alleles) are codominant. When a gene is recessive, its effect (or lack of effect) is masked by a
corresponding dominant gene. The child has a cholesterol level of 7.0 mg/mL. This is in
the severely affected range according to the passage. This child appears to have acquired
one HC gene from each parent for a total complement of two HC genes and no normal
genes. This is consistent with the idea that the normal and HC genes are co-dominant.
Therefore, the best answer is answer choice B.
160. If the lacteals of the intestinal villi were to contract, it would reduce fat absorption

because ingested fats collect in the lacteals for transport to the venous (portal) circulation.
The correct answer therefore is answer choice A. If the capillaries of the intestinal
smooth muscle constricted, the rhythmic contractions of the intestines would be reduced,
but there would be no specific effect on fat absorption. Any constriction of blood vessels
associated with the peritoneal covering of the intestine would have no effect on fat
absorption, because this covering is on the outside of the organ, opposite the luminal
surface where fat absorption takes place.
161. The strongest support for the hypothesis that HC has a genetic rather than an

environmental origin comes from the fact that it develops independent of diet. Answer
choice B is the correct statement: that HC develops when diet is held steady. Answer
choice A does not address the question of whether affected and unaffected families have
different diets. Answer choices C and D address issues that do not speak to the question
of whether the disorder is genetic or environmental.
162. According to the stem, in pea plants "short" (t) is recessive to "tall" (T). In a cross of

heterozygotes (Tt x Tt) one would expect that 75% would be "tall" (Tt or TT) and 25%
Page 54 of 64

would be "short" (tt). Given that 787 plants in the next generation were "tall," the closest
prediction of the number of small plants is B, 277.
163. Breakdown of glucose proceeds first by glycolysis, then by oxidation in the citric acid

(Krebs or tricarboxylic acid) cycle. The enzymes for the former process are located in the
cytoplasm and those for the latter are in the matrix of mitochondria. The 14C label,
therefore, would first appear in the cytoplasm, then in the mitochondria, as stated in
answer choice D.
164. The "central dogma" of molecular biology is that DNA encodes RNA, which encodes

proteins. An early (pre-Watson and Crick) version of the dogma was "one gene, one
enzyme." While this question appears to be asking about blood types, the answer is best
discovered by relying on the central dogma. Answer choices A and B can be eliminated
because neither RNA nor genes code directly for carbohydrates. Answer choice D can be
eliminated because whether genes contain carbohydrates or not is irrelevant to the
outcome of protein manufacture. The only answer choice that is consistent with the
central dogma of molecular biology is answer choice C: that the gene codes for an
enzyme that is responsible for attaching the appropriate carbohydrate. People with blood
type B and AB have an enzyme that people with other blood types do not have. The
enzyme adds an extra galactose monosaccharide to the O glycolipid expressed on the red
blood cell surface.
165. In starvation, the body uses up its stores of carbohydrate and lipids, and then begins to

break down body proteins for metabolic energy. A byproduct of the metabolism of the
amino acids from protein is nitrogen. The correct answer is therefore answer choice B:
the nitrogen in the urine comes from breakdown of the body's proteins. Neither lipids, nor
carbohydrates such as glycogen, contain nitrogen, so the excess nitrogen could not come
from those sources. The reason animals have kidneys is to provide a way of eliminating
nitrogenous waste products. Incomplete reabsorption of nitrogenous products is the goal
of the kidney, not necessarily a pathological process.
166. The primitive eukaryotic cells are described in the passage as anaerobes, so it is likely

that they could breakdown sugars by glycolysis. That means answer choice A is
incorrect. The bacteria are described as aerobic, so the passage suggests that the ability to
use oxygen was acquired with the bacteria. Electron transport in mitochondria and
bacteria is described and the electron transport system is needed for eukaryotes to use
oxygen. The acquisition of aerobic bacteria as partners would have provided them with
the ability to carry out aerobic metabolism through use of the Krebs cycle and electron
transport. Answer choice B is therefore the correct choice. The anaerobic, eukaryoteprecursor cells must have engaged in cell division and the processes of transcription and
translation, so answer choices C and D are incorrect.
167. The endosymbiotic theory described in the passage is widely accepted today, so there

must be some way of using the theory to reconcile the fact that mitochondrial proteins are
made in the cytoplasm. We know, for example, that lateral transfer of genes from one
genome to another is widespread among living organisms. It would have been an easy
matter for mitochondrial genes to be acquired by the host cell nucleus and to disappear
Page 55 of 64

from the mitochondrial genome. It is likely that this exchange would have resulted in a
net gain in efficiency for the composite organism. Answer choice A is therefore the most
plausible answer. Despite many similarities between mitochondria and bacteria, the
question of cytoplasmic synthesis of mitochondrial proteins is relevant to the question of
the origin of mitochondria and needs to be answered. The fact that mitochondria do have
all the machinery (ribosomes, t-RNA, etc.) for protein synthesis eliminates answer choice
C from consideration.
+

168. Hydrogen ions (H ) are protons. The provision of a channel for proton flow across the

membrane would allow hydrogen ions to flow across the membrane until equilibrium had
been achieved between the concentrations on each side of the membrane. Because ATP
synthesis is driven by a flow of hydrogen ions down a concentration gradient, ATP
production will decrease and eventually stop as equilibrium is established (not increase as
suggested in answer choice A or remain unchanged as suggested in answer choice D).
The decrease has nothing to do with the rate of hydrogen ion donation by NADH, answer
choice B. Answer choice C is the correct answer.
169. When a eukaryotic cell ingests a particle in the process known as endocytosis, the

particle ends up in the cytoplasm inside a vesicle, a hollow membrane bounded


intracellular organelle. The membrane of the vesicle is formed from a closed-up patch of
membrane detached from the cell's own plasma membrane. Prokaryotes also have their
own plasma membranes. It seems likely that the two membranes of the mitochondrion
consist of an inner prokaryote-derived membrane and an outer eukaryote-derived one,
this arrangement having been established when some early cell engulfed a bacterium
through the process of endocytosis. Answer choice C describes this arrangement best.
Because the primitive eukaryotic-precursor cell did not use electron transport, their
membranes lacked the complicated protein machinery for this process. The inner
membrane of mitochondria contains this machinery while the outer one lacks it. Answer
choices B and D both suggest eukaryotic membranes make up the inner membrane, so
they must be wrong. If answer choice A was true, the electron transport proteins would
be found in both membranes of mitochondria.
170. Answer choice D, is the correct answer. It presents an example of bacteria inhabiting

eukaryotic cells that is analogous to the hypothesized relationship between mitochondria


and their eukaryote hosts. Because it shows that such a relationship is possible, it
provides stronger evidence than the other answer choices in support of the theory
presented in the passage. That the number of genes in a mitochondrion is less than that of
the typical bacterial cell (answer A) does not support the hypothesis in the passage, it
refutes it. That mitochondria have hundreds of different enzymes (answer B) neither
supports the theory or refutes it since the number in bacteria and mitochondria are not
compared. That mitochondria have diameters that are the same as bacteria is dubious
(answer C) since both mitochondria and bacteria are very diverse in their sizes and
shapes.
171. Answer choice C is the correct answer because it states that the fact that mitochondria

and bacteria both have circular DNA is a reason to think they are related. Such a
characteristic is likely to have been highly conserved over time since it involves the basic
Page 56 of 64

material of life. Changes in the topology of the DNA molecule would involve substantial
changes in the way the molecule replicated and the way it was transcribed. The fact that
both have circular DNA supports the symbiotic hypothesis presented in the passage.
Alternative choices A and B presented are not actually true characteristics of bacteria.
Bacteria do have 70S ribosomes, not the 80S ribosomes of eukaryotes as stated in answer
choice A and they do reproduce by binary fission, which choice B denies. The loss of
ability to carry out anaerobic respiration (answer D) might be a product of the long
history of association with the host cell. It is not unusual for symbiotic organisms to lose
abilities that are compensated for by host functions.
172. Any disruption of mitochondria is likely to decrease ATP production since they are a

major cellular source of that molecule. Answer choices C and D can not be right because
they propose an increase in ATP production. There is no information in the question to
suggest that valinomycin will cause K+ to compete with H+ for an active site on ATP
synthetase. Furthermore, one would suspect that ATP or precursor molecules such as
phosphate and ADP would occupy the active site on the ATP synthase molecule. Answer
choice A, therefore, does not seem plausible. The question does provide the information
that valinomycin increases the flow of K+ across the membrane. An influx of another
positively charged ion into the compartment would disrupt the electrochemical gradient
responsible for the necessary flow of protons. Answer B, therefore, seems more plausible
than any other choice.
173. The carbonyl group (C=O) in carvone makes carvone more polar and has a higher

boiling point than the hydrocarbon limonene. Thus, answer choice A is the best answer.
174. The passage mentions that the ebulliator was added to introduce small air bubbles into

the system. This is the same function provided by a boiling chip at atmospheric pressure.
The air bubbles break the surface tension of the liquid being heated and prevent
superheating and bumping. Thus, answer choice C is the best answer.
175. We presume that the separation of limonene and (+)-carvone can be improved by

modifying either the apparatus shown in Figure 1 or the procedure (i.e., the separation as
described in the passage did not achieve a totally satisfactory separation.) The separation
of the two liquids takes place in the fractionating column as the two liquids vaporize and
condense, with the lower-boiling liquid distilling first. If the fractionating column is
shortened (Answer D), the liquids will vaporize and condense fewer times (i.e., there will
be fewer theoretical plates) and the degree of separation will worsen. Cooling the
condenser with ice water (Answer C) will have no effect on the degree of separation
because condensation takes place after the separation has occurred. Creating a lower
pressure inside the distilling apparatus (Answer B) will lower the boiling points of both
liquids and narrow their difference in boiling point, making it necessary to also increase
the length of the fractionating column in order to achieve the same degree of separation
as obtained originally. Lengthening the fractionating column was not part of answer
choice B. Heating the distillation flask (i.e., increasing the temperature) at a slower rate
(Answer A) will allow both liquids more time in the fractionating column (increase the
number of theoretical plates, allowing liquid and vapor to equilibrate); therefore, of the

Page 57 of 64

four options, this one is most likely to improve the degree of separation of the two
compounds. Thus, answer choice A is the best answer.
176. Carbon 5 is the only stereogenic carbon atom (chirality center); carbons 2 and 7 are not

stereocenters. Thus, answer choice B is the best answer.


177. The boiling point of a liquid is the temperature at which the vapor pressure of the liquid

equals the surface pressure. The normal boiling point is measured at 1 atm pressure. The
vapor pressure of a liquid increases with increasing temperature. Hence, the boiling point
of a liquid decreases as the pressure on the surface of the liquid is decreased. If a leak
develops in the apparatus, the surface pressure will increase, as will the boiling points of
both liquids. Thus, answer choice A is the best answer.
178. To solve this genetics problem, we will take a systematic five-step approach. First, we
need to assign symbols to the genes involved. The question asks us to assume that the
defective hormone response is X-linked. So, symbolize the gene that causes a lack of
response to the X hormone as XX and the normal (wild type) gene as just X. The second
step is to list the possible types of haploid gametes the father can produce and the
possible types the mother can produce. Since the gene is on the X chromosome, we will
leave out all chromosomes except the sex chromosomes X and Y. The father is affected
in this case. He must therefore have the XX gene. The father's genotype is therefore XXY.
No information is given about the mother, but since the trait is very rare, chances are the
mother does not carry the defective gene. We will assume that the mother's genotype is
XX. The third step is to enter the relevant maternal and paternal haploid gamete
genotypes into a Punnett square, putting XX and Y across the top and X and X down the
side. These are the all the possible haploid genotypes of the father's spermatozoa and the
mother's ova. The fourth step is to fill in that square with all possible combinations of
those gamete genotypes that would be produced by fertilization. The fifth step is to
examine the genotypes in the squares and determine which would express the genetic
disorder, that is, which would be unable to respond to hormone X. Since the passage tells
us that no females are affected, the XXX genotype does not express the trait. From the
diagram therefore it appears that none of the offspring, male or female, will be affected.
This is the common result in many X-linked disorders such as hemophilia. Males
predominantly have the disorder, but do not pass it on to their offspring. Females rarely
have the disorder, but can be carriers.

Contribution
of father's
gametes

Contribution

XX

XXX

XY

Page 58 of 64

of mother's
X
gametes

XXX

XY

179. The passage tells us in statement 4 that protein R is capable of phosphorylating Protein

P. Phosphorylation is the addition of high-energy phosphate groups by a kinase to another


protein. A molecule such as ATP (adenosine triphosphate) donates the phosphate group.
The only protein that utilizes ATP is protein R (answer B). Protein R must split ATP,
releasing ADP (adenosine diphosphate) and attaching a phosphate molecule to Protein P.
Thus, answer choice B is the best answer.
180. Because phosphorylated Protein P was necessary and sufficient to increase the rate of

Ca2+ into the cell (statement 6 in the passage), Protein P is probably a membrane channel,
a pore in the membrane that allows Ca2+ to pass freely. Thus, answer choice C is the best
answer.

181. The phosphorylation of Protein P must alter it in some way that allows it to increase

Ca+2 influx. The most immediate way to stop the influx of Ca2+ therefore should be to
dephosphorylate Protein P (answer D). Ca2+ would continue for a time in all the other
answer choices A-C.

182. According to statement 5, Protein P from affected males from Family 2 could not be

phosphorylated. This means that the genetic defect in Family 2 must be in the gene that
codes for Protein P. Therefore, the correct answer choice is B, that the alteration of one
amino acid in Protein P is the likely result of the mutation in the DNA of affected males.
183. If a carbene was the intermediate, then Compound 4 and Compound 5 would form

identical intermediates that would go on to yield identical product distributions. The


passage states that the chemists ruled out Mechanism A because Compounds 4 and 5
formed varying ratios of Compounds 6 and 7. Thus, answer choice C is the best answer.
184. As seen in Reaction 1, OCH2CH3 replaces Br. Thus, answer choice B is the best

answer.

185. By either mechanism, ethoxide abstracts a proton to form ethanol, EtOH, and a bromide

ion is liberated. The spectator ion, Na+, joins with the Br- to form NaBr. Thus, answer
choice A is the best answer.

186. Neither Compound 2 nor Compound 6 is optically active; therefore, the specific

rotation of both compounds is zero. Thus, answer choice D is the best answer.
2

187. Compound 1 contains only sp hybridized carbon atoms and does not contain any sp or
3

sp hybridized carbon atoms. Compound 2 does not contain any sp3 hybridized carbon
atoms but does contain sp and sp2 hybridized carbon atoms. The carboncarbon double
bond that is not part of the two benzene rings in Compound 1 involves sp2 hybrid
orbitals. This double bond undergoes transformation in Reaction 1 resulting in a carbonPage 59 of 64

carbon triple bond in Compound 2. The carboncarbon triple bond involves sp hybrid
orbitals. Thus, Choice A is the best answer.
188. The question asks about the effect of an increase in the level of albumin, one of the

major plasma proteins. Because albumin has nothing to do with the immune response,
answer choice A is incorrect. The plasma proteins can not cross the walls of blood
vessels, but water molecules can. The wall of the artery acts as a semipermeable
membrane setting up the conditions needed for osmosis to occur. An increase in plasma
albumin will upset the osmotic balance because the blood will become hypertonic with
respect to the tissue. Water will have to flow into the bloodstream to reestablish
equilibrium. One of the causes of edema, increased fluid in body tissues, is a decrease in
the plasma protein level. This occurs, for instance, in starvation when the body is forced
to use its albumin as an energy source. An increase in the plasma protein level would
have the opposite effect: fluid would enter the bloodstream (answer D).
189. The taxonomic levels of living organisms, in order, from the groups consisting of the

least closely related to most closely related are: kingdom, phylum, class, order, family,
genus, and species. Distantly related organisms may be in the same kingdom, but as one
descends the hierarchy the taxonomic groups consist of smaller and smaller numbers of
more and more closely related organisms. The question asks which taxonomic level
groups organisms that are the most closely related. The answer is genus, choice B. No
other answer choice is a taxonomic level below genus. There are many mnemonics that
students use to remember the order of the taxonomic levels, such as, "King Phillip came
over from Great Spain." Another way to identify the lowest level of grouping is to recall
that the Linnean binominal nomenclature for a species consists of its genus name and its
species name (Homo sapiens, for instance). Individual species can be uniquely named
using this combination of names because these are the two lowest levels of the taxonomic
series.
190. In imprinting, the perception of an object is enhanced in some way during a critical

period early in development. The term was first used to describe the following behavior
of ducklings, so the correct answer is answer choice A. Habituation, answer choice B, is a
form of non-associative learning in which the response to a stimulus decreases over time,
perhaps due to some kind of neural fatigue. Conditioning, answer choice C, is a form of
associative learning. In classical conditioning, for instance, an animal may come to
associate the sound of a bell with the presentation of food. In operant conditioning, the
performance of an act with the anticipation of a reward, discrimination (answer D) is a
perceptual process, not a learning process.
191. The potential energy of a cycloalkane is a function of its torsional, steric, and ring

strains. In order to make comparisons between cycloalkanes with different numbers of


methylene units, divide the heats of combustion by the number of methylene units in the
cylcoalkane to get the potential energy per methylene. The potential energy of
cyclohexane per methylene is at a minimum when compared to the other cycloalkanes,
because it can adopt a conformation (i.e., a chair) that minimizes the torsional, steric, and
ring strains. When it undergoes combustion, cyclohexane yields less heat per methylene

Page 60 of 64

than the other cycloalkanes, because it is initially at a lower potential energy per
methylene. Thus, answer choice C is the best answer.
192. The compound shown is aromatic and contains tertiary amino groups. Amines can act

as bases or nucleophiles because of the lone pair of electrons on the nitrogen of the amino
group. However, the nitrogens of the tertiary amino groups in the compound shown are
sterically hindered. As a result this compound is not a good nucleophile. Thus, Choice D
is the best answer.
193. The glomerulus is a tuft of capillaries that bulges into the capsular space (also referred

to in some texts by its eponym, Bowman's space), which is a potential space lined with
simple squamous epithelium. Fluid is expressed from the blood across the capillary
endothelium. It enters first the capsular space, then the kidney tubule. The part of the
tubule closest to the glomerulus is the proximal tubule, the next part is the U-shaped loop
of Henle and the last part of the tubule is the distal tubule. Suppose one gradually
increased the rate at which fluid was expressed from the bloodstream (the glomerular
filtrate rate) and measured the concentration of some substance reabsorbed by the kidney
as it left the distal tubule as urine. The concentration might not rise at first, because the
cells lining the tubule might be completely reabsorbing the substance and putting it back
into the blood stream. Eventually, however, the rate of flow would reach a point at which
it exceeds the rate at which the tubule cells could reabsorb the substance. The fluid would
be flowing too rapidly through the tubule for the cells to reabsorb all the substance. The
rate of flow through the tubule at which the substance begins to be observed in the urine
is Tm. At that point the rate of flow of fluid through the tubule begins to exceed the
capacity of the kidney tubule cells to reabsorb the substance. This description of Tm is
given only in answer choice C, the correct answer.
194. The amount of glucose in the fluid in the capsular space as it enters the tubule system

would be the same as the amount of glucose in the plasma, but by the time the fluid
reaches the distal tubule much of the glucose will have been reabsorbed. If the amount
being filtered per minute is less than Tm, all of it will have been reabsorbed. If it is more
than Tm, then the amount will be linearly related to the amount in the plasma. In the case
proposed in this question, the amount being filtered per minute (125mg/min) is less than
the Tm for glucose given in the passage (320 mg/min), so all of the glucose should be
reabsorbed. The correct answer, therefore, is 0 mg/min, answer choice A.
195. Low blood pressure could have an effect on ADH levels (as suggested in answer

choices B and D), which could, in turn, affect the amount of substance A in the urine, but
there is not enough information in the passage to decide if such an effect exists and, if it
does, whether its effect would overcome the certain affect of lowering the blood pressure.
The best answer is answer choice C: that low blood pressure decreases the glomerular
filtration rate, allowing more time for reabsorption and decreasing the amount of
substance A in the urine. Blood pressure is the source of the energy that forces fluid into
the capsular space. If the heart stopped and the blood in the glomerular capillaries had no
hydrostatic pressure, fluid in the space around the glomerulus would flow back into the
capillary bloodstream. This would occur because the protein-rich blood would be

Page 61 of 64

hypertonic with respect to the protein-poor fluid in the capsular space so that the fluid
would flow down the osmotic gradient into the blood.
196. The Tm is a characteristic of the individual substances in the tubule system and a

measure of how efficiently each substance can be reabsorbed. A high Tm indicates a high
capacity for reabsorption of substances in the kidney tubules. In figure 1, the Tm for each
substance can be read as the concentration in plasma when the concentration in the urine
is zero. In this case it looks like the Tm for Substance A is a little over 6 mg/mL and that
of glucose is 10 mg/mL. So substance A has a lower Tm. This means the tubules will not
reabsorb it very efficiently. Much of it will be spilling into the urine, thus being
eliminated from the body. The question asks which will clear from the blood more
rapidly at a concentration of 8 mg/mL and why. The answer is that all glucose will be
reabsorbed at that concentration, none will appear in the urine and none will be cleared
from the plasma. Glucose has such a high Tm, that all of the glucose will be reabsorbed
into the bloodstream, perhaps to reenter the kidney tubule again. A rate of 8mg/mL is
above the Tm of substance A, so there will be some substance A in the urine at this
plasma concentration. The answer to the question depends on the value of Tm, not the
slope of the clearance line, so answer choices A and D can be eliminated. Substance A
will clear more rapidly than glucose, therefore, answer choice B is correct.

197. Above 10 mg/mL, glucose begins to be found in the urine. The Tm for glucose is

therefore 10 mg/mL, answer choice B. This can be read from the graph by looking at the
concentration in the plasma where the concentration in the urine is zero. In other words,
where the clearance line for glucose crosses the axis.

198. The best answer is that increased blood pressure will affect the glomerular filtration

rate, answer choice A. Tm is a characteristic that depends on the characteristics of the


cells lining the renal tubules and independent of blood pressure, so answer choice B is not
correct. Water resorption and concentrating ability are the same, so answer choices C and
D are essentially the same. Increasing blood pressure should increase flow of fluid
through the kidney system and decrease, rather than increase, water reabsorption, so these
answer choices are incorrect.
199. The glomerulus functions as a filter, removing fluid from the capillary blood and

leaving behind molecules and formed elements too large to pass through. This fluid is
called glomerular filtrate. The capsular space is lined with simple squamous cells
unsuited for reabsorption, so answer choice A is incorrect. The first reabsorptive segment
of the tubule is the proximal tubule. Most of the glucose should be reabsorbed in the
proximal tubule since it is the first segment to process the filtrate. In fact, normally, all
glucose reabsorption takes place in the proximal tubule (which has been calculated to
remove half a pound of glucose a day from the glomerular filtrate). The proximal tubule
is lined with simple columnar cells with a brush border on the luminal side consisting of
microvilli. The microvilli give the cell an extensive surface area in contact with the
filtrate to facilitate reabsorption. Therefore, the correct answer is answer choice B.
200. The correct answer is answer choice A: that women have lower bone density than men.

Men and women have the same number of vertebrae, and there is no evidence presented
Page 62 of 64

in the passage to suggest that there are sex differences in calcium uptake or vitamin D
production. Lower initial bone density would increase the impact of any subsequent bone
loss.
201. Estrogen and progesterone are actively secreted by the ovaries of pre-menopausal

women and act to maintain the uterine cycle. With advancing age the ovary becomes less
responsive to pituitary gonadotropins and cyclical changes in the endometrium of the
uterus disappear. The menstrual cycle can be re-established by administration of estrogen
and progesterone in a regimen that approximates the rise and fall of hormone levels in
pre-menopausal women. So the correct answer is answer choice C, periodic menstruation
will resume. To the extent that the therapeutic dosages are typical of pre-menopausal
women, side effects such as breast tissue atrophy (answer A), vaginal tissue drying
(answer B) and lactation (answer D) would not be expected to occur.
202. Calcium levels in the blood need to be kept constant. Parathyroid hormone and

calcitonin regulate blood levels of calcium. The passage gave the information that
calcitonin analogs inhibit osteoporosis. If calcitonin inhibits osteoporosis, it must
function to take calcium out of the blood and into the bone while preventing the loss of
calcium from bone into the blood. High levels of blood calcium available for deposit into
bone should stimulate this process rather than inhibit it. Thus answer choice B, calcitonin,
must be incorrect. Because growth hormone (answer A) and thyroid hormone (answer C)
are not involved in the regulation of blood calcium levels, they are also incorrect. The
correct answer must be parathyroid hormone (answer D) which is inhibited by high levels
of calcium. A mnemonic students use to remember which hormone puts calcium into
bone and which gets rid of it is: Calcitonin-in, parathroidrid.
203. The passage only states that post-menopausal women show the accelerated bone loss

and the acceleration slows after eight to ten years. From this it seems likely that the
acceleration has something to do with withdrawal of estrogen. Administration of estrogen
to men would then have no therapeutic effect on bone loss, since they should not be
suffering from withdrawal of the hormone, having never had high levels. Men given
estrogen should have the same chance of developing osteoporosis as a control population,
answer choice C.
204. Osteoporosis involves two factors, decreased formation and increased breakdown of

bone. Osteoblasts mediate the former process and osteoclasts the latter. Decreased
osteoblast activity and increased osteoclast activity result in bone loss, answer choice D.
205. The question states that the product of the aldol self-condensation is a -hydroxy

ketone, meaning that the condensation is not complete, in that the product has not lost
water. (Note: An aldol addition reaction forms a -hydroxycarbonyl compound; whereas,
an aldol condensation reaction implies that the addition product has lost water.) Because
the addition product is a dimer of the starting compound, the product has a molar mass
that is twice that of the starting compound. In this case, the molecular weight of the
starting compound is 144/2 = 72 g/mol = Answer B.

Page 63 of 64

206. This question involves a reaction at equilibrium. Thus, Le Chtelier's principle is in

effect, and the position of equilibrium can be shifted toward Product A by removing it as
it forms. Catalysts (Answers A and B) do not affect the position of equilibrium, and
heating acetone to the boiling point (Answer D) would remove reactant and shift the
equilibrium to the left. Thus, answer choice C is the best answer.
207. The compound at answer choice D is a constitutional isomer of Product A; the other

compounds are not. Thus, answer choice D is the best answer.


208. Product B contains a double bond, because it results from the dehydration of Product A.

Thus, the addition of a drop of Br2/CCl4 to Product B will result in the red color of
bromine disappearing as bromine adds to the double bond. Thus, answer choice B is the
best answer.
1

209. The six hydrogen atoms in acetone are magnetically equivalent and their H NMR

signal appears as a singlet near = 2 ppm. Thus, answer choice A is the best answer.
210. All three compounds are methyl ketones. The passage states that Product A gives a

positive iodoform test, so we can conclude that all three compounds give a positive
iodoform test. Thus, answer choice D is the best answer.

Page 64 of 64

You might also like